California
Bar
Examination
1
Essay Questions
and
Selected Answers
July 2012
ESSAY QUESTIONS AND SELECTED ANSWERS
JULY 2012
CALIFORNIA BAR EXAMINATION
This publication contains the six essay questions from the July 2012 California Bar
Examination and two answers to each question that were written by actual applicants
who passed the examination after one read.
The selected answers were assigned good grades and were transcribed for publication
as submitted, except that minor corrections in spelling and punctuation were made for
ease in reading. The answers are reproduced here with the consent of their authors.
Question Number
2
Contents Page
1 Civil Procedure 4
2 Community Property/Professional Responsibility 22
3 Evidence 40
4 Contracts 58
5 Wills and Succession 71
6 Criminal Law and Procedure 85
JULY 2012
ESSAY QUESTIONS
3
California
Bar
Examination
Your answer should demonstrate your
ability to analyze the facts in question, to
tell the difference between material and
immaterial facts, and to discern the
points of law and fact upon which the
case turns. Your answer should show
that you know and understand the
pertinent principles and theories of law,
their qualifications and limitations, and
their relationships to each other.
Your answer should evidence your
ability to apply law to the given facts and
to reason in a logical, lawyer-like
manner from the premises you adopt to
a sound conclusion. Do not merely
show that you remember legal
principles. Instead, try to demonstrate
your proficiency in using and applying
them.
If your answer contains only a statement
of your conclusions, you will receive little
credit. State fully the reasons that
support your conclusions, and discuss
all points thoroughly.
Your answer should be complete, but
you should not volunteer information or
discuss legal doctrines which are not
pertinent to the solution of the problem.
Unless a question expressly asks you to
use California law, you should answer
according to legal theories and
principles of general application.
Question 1
Pam and Patrick are residents of State A. While visiting State B, they were hit by a
truck owned and operated by Corporation, a freight business.
Corporation is incorporated under the laws of Canada and has its headquarters there,
where its President and Secretary are located. State B is the only state in which
Corporation conducts its business. Corporation’s drivers and other employees work out
of its warehouse in State B.
Pam and Patrick jointly filed a lawsuit against Corporation in federal district court in
State A. In their complaint, Pam demanded damages for personal injury in the amount
of $70,000 and for property damage in the amount of $10,000; Patrick demanded
damages in the amount of $6,000.
Corporation filed a motion to dismiss the complaint for lack of personal jurisdiction. The
federal district court denied the motion. After trial, it entered judgment for Pam in the
amount of $60,000 and for Patrick in the amount of $4,000.
Corporation has appealed on the grounds of lack of subject matter jurisdiction and lack
of personal jurisdiction.
How should the court of appeals rule on each ground? Discuss.
4
ANSWER A TO QUESTION 1
1. Did the Federal District Court of State A have Personal Jurisdiction over
5
Corporation?
Waiver?
Personal Jurisdiction is waived if not challenged. Here, Corporation ("Corp") filed a
motion to dismiss for lack of personal jurisdiction ("PJ") at trial. Therefore, Corp did not
waive its right to appeal based on lack of PJ.
Personal Jurisdiction
Personal jurisdiction is the power of a court to have jurisdiction over an individual or
entity. Here, a corporation. The exercise of personal jurisdiction must comport with the
requirements of Due Process.
TRADITIONAL BASIS
Traditionally, PJ could only be exercised if the defendant consented to suit in the forum,
was served in the forum, or was domiciled in the forum. Here, there are no facts to
indicate that Corp consented to jurisdiction ("JDX") because they did not make a
general appearance, or in any way consent. Further, Corp is not domiciled in the forum.
A corp is domiciled where it has its principal place of business, based on nerve center,
and where it is incorporated. Both of those are in Canada for Corp. Finally, facts do not
state where Corp was served or if they had an agent for service of process in State A,
but assuming that they were not served in state A, there is no traditional basis in state
A.
LONG ARM STATUTE
If there is no traditional basis for the exercise of Personal jurisdiction, the court will next
look to the state's long-arm statute to determine whether the court has jurisdiction to
reach out to another state, or country to exercise jdx over the defendant. Here, there
are no facts to indicate that state A has a long-arm. If it did, the federal district court
would have jdx to the same extent as the state.
CONSTITUTIONAL ANALYSIS
6
To comport with due process, personal jurisdiction is only proper if the defendant has
such minimum contacts with the forum state that the exercise of jdx comports with
traditional notions of fair play and substantial justice.
MINIMUM CONTACTS
Minimum contacts requires a showing of purposeful availment and foreseeability.
PURPOSEFUL AVAILMENT
A party purposefully avails itself of the forum state if it has taken advantage of the
benefits and protections of that state's laws. Here, Corporation is incorporated in
Canada and has its headquarters there. Further, its warehouse is in state B. Further,
Corp oprates a freight business and was driving in state B when the Accident occurred,
and state B is the only state in which Corp conducts its business. There are no facts to
show that Corporation had any contact at all with state A. Therefore Corp will argue it
did not purposefully avail itself of the privileges and benefits of State A.
Foreseeability.
Because State B is the only state in which corp does business, it will argue that it was
not foreseeable that it would be haled into court in state A. P and P could argue that a
trucking company should foresee being sued anywhere, but if the trucks are only on the
road in state B, this argument will not likely prevail. It was not foreseeable that Corp
would be sued in state A.
Relatedness of the claim to the contact
The court will look at the quality and nature of the contacts. There is general jurisdiction
if the defendant's contact is so systematic and continuous that he is essentially at home
in the forum. There is specific jurisdiction if the contact is less than systematic and
continuous, but the claim arises out of the defendant's contact with the forum.
Here, there is neither general or specific jdx because the claim neither arises out of the
contact with the forum nor is Corp "essentially at home" in the state A because its
contact there is not systematic and continuous.
Specifically, the accident occurred while Pam and Patrick were visiting in State B, not
state A, and therefore the claim does not arise out of contact with State A, and there
can be no Specific Jdx as a result.
Additionally, Corp only does business in State B, has its warehouse in state B and is
incorporated and has its president, secretary and headquarters in Canada. Therefore
there is no general jdx because there is no contact with State, and certainly not
systematic and continuous contact.
Therefore, there is neither specific nor general Jurisdiction.
Fairness
7
The fairness factors include the convenience of the parties and witnesses as well as the
forum state's interest. The court will also look at the interstate judicial system's interest.
Although state A has an interest in providing a forum for redress for its citizens, and
Pam and Patrick are State A citizens, State B has a stronger interest because that is
where the accident occurred, on its roads. Further, because Corp operates in State B
only, state B has a strong interest in adjudicating the claims against its citizens for their
conduct while in the state. As to convenience, any and all witnesses and evidence
would be located in State B, rather than the forum, state A.
Therefore, the fairness factors are in favor of not finding PJ over Corp.
RULING:
8
Therefore, Under a Due Process Analysis, The court of appeals should rule that there
was no personal Jurisdiction over Corp.
2. Did the Federal District Court have Subject Matter Jurisdiction over the matter?
Federal courts are courts of limited jdx and must have jurisdiction under arising
under/federal question jurisdiction or diversity jurisdiction.
In some cases, the court may be able to exercise supplemental jurisdiction.
Federal District Court must have had jurisdiction over each and every claim in the
matter. Here, both Pam and Patrick brought claims. Therefore, each claim is
considered separately below.
WAIVER?
Here, it does not appear that Corporation contested subject matter at trial. However,
subject matter is not waived if the party fails to raise it at trial, and may be raised at any
time, even on appeal. Therefore, Corporation could appeal based on this ground.
SUBJECT MATTER JURISDICTION OVER PAM'S CLAIM
May have SMJ under either federal question or diversity. Here, the claim arises from
personal injury, a tort claim, which is a state claim. Therefore, Pam must show diversity
jurisdiction.
Diversity jurisdiction requires complete diversity between the parties (Strawbridge v.
Curtiss) and an amount in controversy in excess of $75,000.
COMPLETE DIVERSITY
Complete diversity requires that all plaintiffs are citizens of different states than all
defendants. Or, as in the case here, that the suit be between a citizen of a state, and a
foreign citizen.
A natural person is a citizen of the state in which she is domiciled. Domicile is physical
presence plus intent to remain indefinitely. Here, facts state that Pam is Resident of
State A. Therefore, Pam will be domiciled in state A.
A corporation is a dual citizen of every state in which it is incorporated and the state
which is its principal place of business. ("PPB") PPB is determined by the "nerve
center," or the place from which corporate managers run the corporation. (Hertz v.
Friend).
Here, Corp is incorporated under the laws of Canada and thus is a citizen of Canada.
Further, Corp has its "nerve center" in Canada because that is where its headquarters is
located and where its President and Secretary are located. Although Corp keeps a
warehouse in state B and its drivers and other employees work out of the warehouse in
state B, no facts indicate that any direction of corporate activity occurs here. Therefore,
this is the muscle center, not the nerve center, and the Supreme Court ruled that the
Nerve center is the PPB.
Therefore, Corp will be deemed a citizen of Canada, meaning that it is a foreign citizen.
Because Pam is a citizen of State A and Corp is a Foreign Citizen of Canada, there is
complete diversity between the parties.
AMOUNT IN CONTROVERSY
9
The amount in controversy must exceed $75,000 exclusive of interests and cost. The
plaintiff's good faith claim will control, unless it is clear to a legal certainty that plaintiff
cannot recover the required amount (in excess of $75,000).
Here, Pam demanded damages of $70,00 for personal injury and $10,000 for property
damages. Neither amount alone satisfies the amount in controversy.
AGGREGATION
Generally, aggregating claims is not required. However, a single plaintiff may
aggregate all claims against a single defendant. This means that Pam can add together
her claims against Corp. Therefore, adding Pam's claims together, her good faith claim
was for $80,000. Because there are no facts to indicate this amount was not in good
faith, or that there is a legal certainty prohibiting Pam from this recovery (such as a
statutory damages cap), Pam has met the amount in controversy.
RESULT IF PLAINTIFF RECOVERS LESS THAN THE AMOUNT IN CONTROVERSY:
10
If the plaintiff recovers less than the amount in controversy, that will not defeat diversity
jdx, because the good faith claim controls. However, in such a case, the plaintiff may
be required to pay the defendant's fees in the litigation. Therefore, because Pam
recovered on $60,000 that will not defeat diversity, but she may be liable for costs.
RULING:
The federal district court had subject matter jurisdiction over Pam's claim by virtue of
diversity jurisdiction. Therefore, the court should deny the appeal based on lack of SMJ
over Pam's claim.
SUBJECT MATTER JURISDICTION OVER PATRICK'S CLAIM
As above, this is a tort claim, not arising under federal law, and therefore the court will
not have "federal question" jurisdiction. Therefore, Patrick will have to meet the
requirements of diversity jurisdiction for the federal district court to have had SMJ.
COMPLETE DIVERSITY
Like Pam above, Patrick is domiciled in state A and will therefore be a citizen of state A.
Under the analysis above, Corp is a foreign citizen of Canada. Therefore, as above,
there is complete diversity.
AMOUNT IN CONTROVERSY
Patrick requested only $6000 in damages. This is less than $75,000 and therefore does
not meet the amount in controversy. Patrick may not aggregate his claim together with
Pam, because plaintiffs may not aggregate claims with other plaintiffs.
Therefore, the court did not have diversity jdx over Patrick's claim.
SUPPLEMENTAL JDX
11
Where the court has jurisdiction over one claim in a matter, it may exercise
supplemental jurisdiction over other claims that arise from a Common Nucleus of
Operative Fact. The common nucleus test is generally considered broader than the
same transaction or occurrence test, and therefore any party that would meet the Same
Tran. and Occ. test will meet the Common Nucleus of Operative Fact test.
Here, Pam and Patrick are both suing for injuries and damages arising from the same
car accident. While visiting State B, they were hit by a truck owned by Corp, the same
truck, in the same accident. The witnesses to both will be the same, as will the
evidence. Therefore, Patrick's claim arises from a Common nucleus of operative fact
with Pam's claim, and the federal district court could exercise supplemental jdx over
Patrick's claim.
DIVERSITY LIMITATIONS ON SUPPLEMENTAL JDX
However, where the underlying claim is in diversity, the court cannot exercise
supplemental jdx over a claim by a plaintiff that would defeat complete diversity. Here,
Patrick is a plaintiff. However, if supplemental jdx is exercised over Patrick’s claim it will
not defeat complete diversity because all Plaintiffs will still be citizens of State A, and all
Defendants of Canada.
Where the supplemental claim does not meet the amount in controversy, but will not
destroy complete diversity, the court may exercise supplemental jurisdiction over the
claim. Here, Patrick's claim did not meet the amount in controversy, but will not destroy
complete diversity and therefore the court may exercise supplemental jdx over the
claim.
DISCRETION
In some cases, a federal district court should exercise discretion not to exercise
supplemental jdx, such as where there is a novel or complex issue of state law, or state
claims predominate, or all federal questions have been dismissed. On these facts, this
is a tort claim for personal injury and therefore not novel or complex. Further, the claim
is in diversity and not federal question, and thus there is no concern about the federal
claims being dropped out.
This is not a claim over which the court should decline supplemental based on the
discretionary factors.
RULING
12
The Federal district court had subject matter jurisdiction over Patrick's claim based on
supplemental jurisdiction. Therefore, the appeals court should deny the motion on the
basis of lack of subject matter jurisdiction.
ANSWER B TO QUESTION 1
Pam and Patrick v. Corporation
13
Pam and Patrick have raised a claim against Corporation (C) in federal district court in
State A. Corporation attempted to dismiss the case based on lack of personal
jurisdiction (PJ) and subject matter jurisdiction (SMJ). These motions were denied, Pam
and Patrick were awarded damages in the case, and Corporation has appealed the
case on the grounds listed above. The following considers how the court of appeal
should rule on these claims.
Subject Matter Jurisdiction
Subject matter jurisdiction (SMJ) considers whether the court has the power to hear the
particular case. This case was brought in federal court; federal courts are courts of
limited power, unlike state courts, which can generally hear any case save for several
exclusively federal categories. In order for federal courts to have proper SMJ over a
case, the case must either be based on a federal question, or meet the requirements for
diversity of citizenship jurisdiction. Each of these will be examined in turn to see if the
federal courts have jurisdiction over this matter.
Federal Question
A case may properly be held in federal court when the case is based on a federal
question. This requires that the plaintiff assert a claim arising under the federal
constitution or a federal law. The "well pleaded complaint rule" dictates that the claim be
asserted in plaintiff's complaint. It is not enough that a federal issue generally be raised
by the case, nor that the defendant will defend on the grounds of a federal law.
Here, the case involves personal injury damages for the injuries that Pam and Patrick
suffered when they were hit by a truck owned and operated by Corporation. Thus, it
appears that the case is just a simple tort case, which would be based on state law, and
not on the constitution or federal law.
Thus, there is no federal question here.
Diversity in Citizenship
14
However, the federal courts have another means of jurisdiction available, in the form of
diversity of citizenship. To be valid, all plaintiffs must be "diverse" in citizenship from all
defendants, and the amount in controversy must exceed $75,000.
Diversity
There is an absolute diversity requirement, wherein each plaintiff must be entirely
diverse in citizenship from each defendant. The federal rules allow for diversity between
citizens of different states, or citizens of a state and a foreign country. Citizenship for
individuals is based on their domicile, which is evidenced by physical presence and
intent to remain. For corporations, citizenship is determined by place of incorporation,
and principal place of business, which is where the owners, directors, and management
manage and direct the company's affairs.
Here, Pam and Patrick are residents of State A. Though the facts do not give us any
hints into whether they have the intent to remain there, it is reasonable to presume that
they did have that intent. Thus, their citizenship is State A.
C is incorporated under the laws of Canada and has its headquarters there, where its
President and Secretary are located. Thus, the place of incorporation and the principal
place of business is in Canada. Of note, Corporation's drivers and other employees
work out of its warehouse in State B. Several years ago, this may have met the
"muscle" test, and thus demonstrated citizenship in State B for C; however, this test has
been done away with. Nonetheless, there still would be diversity in citizenship even if C
was a citizen of State B.
Thus, there is diversity in citizenship, because Pam and Patrick are citizens of State A,
and C is a citizen of Canada. Because the rules of civil procedure allow for diversity
between residents of a state and a foreign country, there is proper diversity.
Amount in Controversy
Next, the amount in controversy must exceed $75,000 excluding interest and attorney's
fees. The court will examine this based off of a good faith pleading of damages by the
plaintiff. To reach this amount in controversy, any single plaintiff may aggregate as
many claims together to meet the minimum requirement. However, multiple plaintiffs
may not aggregate claims in order to reach the minimum requirement.
Here, Pam demanded damages for personal injury in the amount of $70,000, and for
property damage in the amount of $10,000. This is an example of aggregation by one
plaintiff against one defendant. This is proper. Further, because $70,000 plus $10,000
equals $80,000, it exceeds the amount in controversy requirement.
Patrick demanded damages in the amount of $6,000. This would not be able to be
aggregated with Pam's claims in order to reach the amount in controversy; however,
because Pam has reached the amount all on the basis of her own claims, this does not
impact Patrick's claim. We will need to examine whether Patrick's claim can be joined,
however. I will do this later under supplemental jurisdiction (see below).
The conclusion is that the amount in controversy is met, as Pam's claims exceed the
required $75,000 minimum amount.
The Effect of Receiving Less Than $75,000 At End Of Trial
15
C may argue that SMJ was invalid because Pam and Patrick ended up receiving less
than $75,000 in damages at the end of the trial. This is incorrect. The mere fact that the
parties recovered less than $75,000 at the end of the trial does not mean that the court
loses jurisdiction, or never had it in the first place. All that is required is a good faith
claim exceeding $75,000. Thus, this will have no effect on the question of SMJ.
Supplemental Jurisdiction: Pat's Claim of $6,000
As discussed above, Pat's claim alone did not meet the amount in controversy
requirement. Each and every claim must meet the requirement in order to satisfy SMJ.
When the amount in controversy is not met, we can look to supplemental jurisdiction to
see if the claim can nonetheless get into federal court.
Supplemental jurisdiction requires that the claim contain a common nucleus of fact with
the other claims asserted. If the claim arises out of the same transaction or occurrence,
then this test is always met. Supplemental jurisdiction cannot be invoked when it would
defeat complete diversity in a diversity case. Here, Pat's claim is based on the exact
same incident as Pam - the accident with C's truck. Thus, it is the same transaction or
occurrence, and will be able to be heard. The federal courts do have discretion to not
hear these claims, but it is likely that they would hear this to get the whole case out of
the way at the same time. Further, adding Pat's claim does not defeat diversity,
because he is a citizen of State A.
Thus, Pat's claim can properly be heard in federal court.
When Can SMJ Be Asserted?
16
Finally, we must consider at what point can SMJ be raised as an issue. Some claims
must be asserted before certain stages of the trial in order to be preserved, and if not
raised, then they are waived. SMJ, however, is never waived, as it is a strict
requirement that the federal courts have subject matter jurisdiction. Thus, it is of no
concern that C apparently has not raised the SMJ issue prior to the appeal; they can still
properly raise it.
Conclusion: The federal court system has proper SMJ over Pam and Patrick's claim.
Personal Jurisdiction
Personal jurisdiction (PJ) considers whether this state can properly hear this claim
against this defendant. It asks whether the state has the power to force the defendant to
come into the state to defend the claim. To examine whether PJ exists over C in State
A, we must look to the traditional bases of exercising jurisdiction, the state long-arm
statute, and to the constitutional limitations on exercising PJ.
Traditional Bases of Exercising Jurisdiction
17
Traditionally, PJ can be asserted over a defendant if the defendant (1) is domiciled in
the state; (2) consents to jurisdiction; or (3) is served with process while in the state. As
discussed above, C is not domiciled in State A, but is rather domiciled in Canada.
Further, it does not appear that C has consented to jurisdiction in State A in any way
(though we will later talk about the need to timely raise the argument of lack of personal
jurisdiction). And finally, there is no indication that C was served in State A.
Thus, the traditional bases of exercising jurisdiction seem to not be present.
Long-Arm Statute
A long-arm statute is a state statute that states when the state can reach and "grab" an
out-of-state defendant, and force the defendant to defend in the state court. Some of
these long-arm statutes require that the defendant commit a tortuous act in the state, or
break a contract in the state, while others simply grant the state the ability to reach out
to grab defendants to the full extent as allowed by the U.S. Constitution.
Here, the facts do not mention the reach of State A's long-arm statute. It is reasonable
to assume that it reaches the constitutional limits. Thus, we must examine the
constitutional limits of PJ.
Constitutional Limitations
To exert PJ over an out-of-state defendant, the constitution requires certain minimum
contacts with the forum state such that maintenance of the suit there does not offend
traditional notions of fair play and substantial justice. To determine if this is true in this
case, we can break the above test down into three sections: minimum contacts,
relatedness of the claim to the contact, and fairness.
Minimum Contacts
The constitutional requires the defendant to have some minimum contacts with the
forum state in order for the state to exert jurisdiction. The defendant must have
purposefully availed himself in the state, such that being subject to a claim in that state
would be foreseeable.
Purposeful Availment
18
Purposeful availment requires that the defendant commit a voluntary act in the forum
state. Defendant must avail himself in some way to the state, whether it be by using the
state's roads, or attempting to make money in the state.
Here, C is incorporated in Canada, and has its principal place of business there. It
conducts business solely in State B, which is also where it has a warehouse. Further,
the accident occurred in State B. It is possible that C drives on State A roads from time
to time, but the facts do not give this information. Also, there are no facts which say that
C ships goods to State A, or otherwise tries to make money there. Simply put, on these
facts, there seem to be no contacts whatsoever with State A, other than that Pam and
Patrick are residents of State A.
The court of appeal should find that there was no purposeful availment.
Foreseeability
The minimum contacts must be sufficient enough to make it foreseeable that defendant
would be "haled into court" in the forum state. Here, as discussed, there appears to be
nothing that C did that would make it foreseeable that they would end up in State A. The
mere fact of driving on State B's roads does not make it foreseeable that they would end
up in State A's court. I suppose if State A were located directly adjacent to State B that it
would perhaps be more foreseeable, but again, the facts do not share that information.
A case against C in State A court was not foreseeable.
Relatedness of the Claim to the Contact
The more related the claim is to the contact with the forum state, the more likely the
court will be to allow for jurisdiction over the defendant.
If the claim arises directly out of the contact with the forum state, this gives rise to
specific personal jurisdiction. Here, there was no contact with State A, and so there
cannot be specific jurisdiction.
Next, general personal jurisdiction may exist if the defendant consistently and regularly
conducts activity in the forum state, such that he is "essentially at home there." Merely
selling goods in a state does not give rise to general PJ, there must be an actual
physical presence. Here, C is not in State A whatsoever, or so it seems. Thus, it is not
essentially at home in State A. It may essentially be at home in State B, where it has a
warehouse, but this does not affect the discussion of whether State A has jurisdiction.
The claim is not related to C's contact with State A, as C has no contact with State A.
Fairness
19
Finally, the court will look to see if holding the suit in the state meets general standards
of fairness. Under this, the court considers convenience to the parties and the
witnesses, the forum state's interests, and the plaintiff's interests.
Convenience
Under the convenience factor, the court will look to see how convenient it is to hold the
case in the forum state, based on a variety of factors including where the parties are,
where the witnesses are, where the evidence is, etc. If the inconvenience to the
defendant grossly impacts his ability to defend against the case, the court will likely
dismiss for lack of PJ.
Here, the accident occurred in State B, so any witnesses are likely in State B. It is
unknown where the wreckage is located, but the vehicles are likely also in State B.
Thus, a good portion of the pertinent materials needed would be in State B. Further, C
has no connection with State A, and will have to travel there to defend against the suit.
This is likely not entirely burdensome, because they are a corporation, and likely would
have the resources to get there.
However, it was likely entirely inconvenient to have the case in State A, based on where
the evidence, witnesses, and the defendant was located.
State's Interests
20
Next, the court will look to see if the forum state has a strong interest in providing a
forum for the claim. Here, State A is interested in providing a forum for its residents; it
wants to be sure that they are compensated for their injuries. However, the accident
occurred on State B's roads, and so State B would have more of an interest, because it
wants to be sure that dangerous drivers are kept off of their roads.
In the end, a court would likely find that State A has a limited interest in holding this
case.
Plaintiff's Interests
Finally, the court looks to the plaintiff's interests in having the case in the forum state. It
is likely that Pam and Patrick have suffered some injuries and thus would prefer to not
have to travel. However, they had already been in State B on vacation, and could likely
travel there again if needed. The court generally will be deferential to the plaintiff's
choice of forum, however.
In the end, it is likely that it is simply not fair to have C defend in State A court.
When Can PJ Be Asserted?
On a final note, PJ must be asserted either in a 12b motion prior to the answer, or along
with the answer. If not, it is waived. Here, it appears that C raised the PJ motion at
some point early on, and thus likely did not waive it, so that it can be heard on appeal.
Some courts require that a party immediately appeal a decision on PJ by way of an
extraordinary writ.
Conclusion:
21
The Court of Appeal should hold the court had SMJ over the matter, but not PJ. Thus,
provided that PJ has not been waived, it should dismiss the case. If it has been waived,
the court should reject the PJ argument as well.
Question 2
Wendy and Hal are married and live in California.
A year ago, Wendy told Hal that she would not tolerate his drinking any longer. She
insisted that he move out of the family home and not return until he completed an
alcohol treatment program. He moved out but did not obtain treatment.
Last month, Hal went on a drinking spree, started driving, and struck a pedestrian.
When Wendy learned of the accident, she told Hal that she wanted a divorce.
Hal has consulted Lawyer about defending him in a civil action filed by the pedestrian.
He is currently unemployed. His only asset is his interest in the family home, which he
and Wendy purchased during their marriage. Lawyer offered to represent Hal if Hal
were to give him a promissory note, secured by a lien on the family home, for his fees.
Hal immediately accepted.
1. Is Wendy’s interest in the family home subject to damages recovered for injuries to
the pedestrian? Discuss. Answer according to California law.
2. Is Wendy’s interest in the family home subject to payment of Hal’s legal fees?
Discuss. Answer according to California Law.
3. What, if any, ethical violations has Lawyer committed? Discuss. Answer according
to California and ABA authorities.
22
ANSWER A TO QUESTION 2
1. Is Wendy's Interest in the Family Home Subject to Damages Recovered for
23
Injuries to the Pedestrian?
California is a Community Property State
California is a community property (CP) jurisdiction. Thus, any property acquired by
either spouse during the course of the marriage by either spouse's labor is
presumptively community property. Property acquired before or after the marriage by
either spouse, or during the marriage by gift, inheritance, or devise, is presumptively
separate property (SP). In determining the character of a particular asset, it is helpful to
look at (1) the source of the asset or the source of the funds used to purchase the
asset, (2) any actions by the spouses changing the character of the property, and (3)
any relevant presumptions.
The House
Source
The facts tell us that Wendy (W) and Hal (H) purchased the family house during their
marriage. However, we don't know what funds were used to purchase the house. If
W's or H's earnings were used (or a combination thereof), and those earnings were
earned during the course of the marriage, then the house would be CP because
spousal earnings are CP to the extent they're earned during the marriage.
However, if one spouse partially used inheritance money or other SP acquired before
the marriage, then that spouse would likely have a SP interest in the home to the extent
SP was used to purchase it.
However, without more, the best assumption is that spousal earnings were used to
purchase the house. The facts say H is currently unemployed, but he may have been
employed in the past (and thus had earnings). Further, we can assume W earned
money somehow, likely from a job.
Actions
24
There is no evidence that the house was put in only one spouse's name, suggesting
that the house was the separate property of that spouse. Pre-1975, if the house was in
W's name, the married woman's special presumption would operate to render the house
(or the share of the house in W's name) W's SP.
Modernly, if title was taken in only one spouse's name, a court would not likely hold that
to be conclusive evidence that the house was that spouse's SP absent some
manifestation by the other spouse that the house was intended as a gift.
If H and W took title to the house as joint tenants with a right of survivorship, each would
have a 1/2 SP undivided interest in the whole during life. On death, the form of title
would control. On divorce, under CA's anti-Lucas statute, the house would be treated
as CP, with a right to reimbursement for any SP used by either spouse to improve the
home.
Finally, there's no evidence of a transmutation changing the character of the house,
which, after 1985, would have to be in writing.
Thus, absent any of these actions, it appears the house is still CP.
Presumption
All property acquired during the course of marriage is presumptively CP. Here, nothing
rebuts that presumption.
Community Responsibility for Debts of One Spouse
All debts incurred by either spouse prior to or during the course of marriage are
community debts. Tort obligations are "incurred" when the tort occurs, not when
judgment is handed down. Thus, any obligations arising out of H striking the pedestrian
were "incurred" when he hit the pedestrian.
W will argue that the marital economic community was not in existence when H hit the
pedestrian because she had kicked him out of the house. The marital economic
community begins at marriage and terminates upon permanent physical separation
when at least one spouse has no intent of continuing the marriage.
Here, W kicked H out of the house. However, she told him that he could return when he
completed an alcohol abuse program. Thus, the marital economic community had not
yet ended when H got in the accident because W was still open to the possibility of him
returning. W will argue that H manifested an intent to never continue the marriage
because he refused to go to treatment. In other words, W will argue that by rejecting
the pre-condition to the continuation of the marriage--i.e. getting treatment--H effectively
terminated the marital economic community. Indeed, W can point to the fact that 11
months after she kicked H out, he hadn't obtained treatment. Given this length of time,
W can argue, it's clear that the community had ended.
However, the stronger argument is that the marital economic community continued until
W told H that she wanted a divorce. If W viewed the marital community as over prior to
the accident, she would have likely filed for divorce then. Instead, it appears the
accident was the "last straw." Thus, the request for a divorce was the clearest signal
by either party that the physical separation was permanent and there was no intent to
continue the marriage.
Thus, the marital economic community had not ended when H struck the pedestrian,
any obligation incurred because of the accident is a community debt.
Order of Payment
25
When a tort is committed during an activity for the benefit of the community, the debt will
be satisfied first by CP, then by the tortfeasors SP. The non-tortfeasor spouse's SP is
not subject to the debt.
When a tort is not committed during an activity for the benefit of the community, the debt
will be satisfied first by the tortfeasor's SP, then by CP. Again, the other spouse's SP is
safe.
Here, H committed the tort against the pedestrian while driving drunk. This was not an
activity for the benefit of the community--to the contrary, H was supposed to be seeking
alcohol abuse treatment while he was living away from the family home. Thus, recovery
would be taken out of H's SP before the CP.
However, on the facts, it doesn't seem as though H has any SP to satisfy the debt.
Thus, any recovery will likely be against the H and W's CP.
Reimbursement to the Community
26
To the extent any CP--i.e. the house--is used to pay any obligation arising out of H's
accident with the pedestrian, the community may be entitled to reimbursement from H.
Where CP is used to pay an obligation arising out of spouse's tort that was committed
not during an activity for the benefit of the community, the community is entitled to
reimbursement for that payment if the tortfeasor's SP was available to pay (or if the
order of payment was not followed). However, as mentioned, it doesn't appear H has
any SP available to pay the debt and, thus, reimbursement may be unlikely.
Distribution of Debts on Divorce
At divorce, community assets are generally divided under the "equal division rule"--i.e.
each spouse gets 1/2 of each community asset in kind.
However, a judge has more discretion as to the allocation of debts at divorce. Typically,
a judge will allocate a tort debt to the tortfeasor spouse if the tort was incurred not
during an activity for the benefit of the community. However, a judge may take into
account ability to pay to effect a more just allocation of debts.
Here, on divorce, the judge would likely allocate any judgment based on H striking the
pedestrian to H. H will argue that he's unemployed and can't pay, but it's highly unlikely
a judge would saddle W with an obligation to pay H's tort liability post-divorce.
Conclusion
Thus, during the marriage, H and W's CP will be liable for damages recovered for
injuries to the pedestrian. Even though H and W have filed for divorce, until community
assets and debts are distributed, the community estate continues and the pedestrian
can recover against it. However, as mentioned, on divorce, the debt will be allocated to
H. Further, W may be entitled to reimbursement for CP used to pay the debt.
*Note: If the court decided that the marital community was terminated when H struck the
pedestrian, then CP--i.e. the house--would not be liable for the debt because the debt
would be H's SP.
2. Is Wendy's Interest in the Family Home Subject to Payment of H's Legal
27
Fees
Equal Management
Each spouse generally has equal rights to manage community property. This includes
the right to sell and encumber community property. However, with respect to real
property, one spouse may not encumber community owned real property without the
other spouse's consent. If one spouse, without consent, sells or encumbers community
real estate, the non-consenting spouse has the power to void that transaction within 1
year.
Lien on the House
Here, H has given Lawyer a lien on the family home without W's consent. Thus, W has
the power within 1 year to void the encumbrance.
H will argue that because he gave the lien on the house after W told him she wanted a
divorce, he was only granting a lien on his 1/2 SP interest in the family home. However,
there's no evidence that W actually filed for divorce or that divorce proceedings were
held during which a judge divided the community estate. While the marital economic
community may no longer exist because there has been permanent physical separation,
the community estate lives on until it has been distributed.
Thus, a court would likely allow W to void the encumbrance on the community real
property due to her lack of consent in making the encumbrance.
Timing of the Attorneys Fees
28
Furthermore, H sought legal advice after W told him she wanted a divorce. Because W
asking for divorce terminated the marital economic community, CP--i.e. the family
home--is not liable for the debts incurred by H after such separation.
Thus, any obligation owed to Lawyer based on legal services rendered to H cannot be
satisfied out of CP because such an obligation would not be a community debt.
He would argue that payment of attorney’s fees is an obligation arising out of the
accident of the pedestrian, when the marital economic community still existed.
However, the attorney’s fees represent an entirely different event. Furthermore,
contractual obligations arise when the contract was made. Here, any contract and/or
agreement with Lawyer was made after the economic community ended. Therefore, W's
interest in the family home is not subject to payment for the additional reason that CP is
not liable for H's separate post-marriage debts.
Necessaries
Post-separation, a spouse can still be liable for obligations relating to necessaries that
the other spouse incurred during the marriage. Necessaries generally refer to food,
shelter, and medical expenses. Here, H's legal fees don't likely constitute necessaries
and, as such, this theory cannot be invoked to hold W's interest in the family home
subject to payment.
3. Lawyer's Ethical Violations
Obtaining Pecuniary Interest in Outcome of Case
Under the ABA, a lawyer cannot obtain a pecuniary interest in the subject matter of a
case other than in the case of a contingency fee arrangement or an attorney's lien.
However, in CA, attorneys' liens are impermissible.
Here, Lawyer effectively acquired an attorney's lien on H's family home. Thus, Lawyer
will argue that this was permissible because the only purpose here was to secure
payment. In CA, this would constitute an ethical violation. Under the ABA, it's less
clear.
While under the ABA, an attorney's lien is permissible, if Lawyer knew that H couldn't
rightfully encumber the family home, then it's possible that Lawyer committed an ethical
violation because accepting the attorney's lien would constitute a violation of a third
party's (W's) rights in the course of representing H.
Entering into Business Transactions with Clients
29
An attorney can only enter a business transaction with a client if (1) the terms are fair
and reasonable, (2) the terms are communicated to the client in an easily
understandable manner, (3) the client is advised to get independent counsel to
represent him in the transaction and is given a chance to do so, and (4) the client
consents.
Here, by taking a lien on H's family home, Lawyer entered into a business transaction
with H. However, it's not clear that Lawyer ever advised H to seek independent counsel
or that he adequately informed him of the material terms of the lien. Although H
immediately accepted, he did so without knowing what would trigger enforcement of the
lien (1 missed payment? total failure to pay? late payment? H's insolvency?). Thus, by
failing to adequately inform H and encouraging him to seek independent advice, Lawyer
likely violated the ethics rules.
Fees
Under the ABA, a fee must be reasonable. In CA, fees can't be unconscionable.
Further, in CA, a fee agreement must be in writing unless it's (1) less than $1k, (2) with
a corporation, or (3) for a routine matter involving an existing client.
Here, the lien agreement was essentially a fee agreement. However, the terms were
not adequately disclosed to H. Further, there was no written fee agreement. Because a
writing was likely required--there's no evidence H was an existing client or that Lawyer's
services were valued at under $1k--this is a violation of CA rules.
Further, the lien was likely unreasonable and unconscionable. Because H was
unemployed, it was extremely unlikely that he was going to be able to pay Lawyer's
fees. If Lawyer knew that H was unemployed--which he likely did, considering he
conditioned representing H on having a lien on the house--then Lawyer must have
known that H wouldn't be able to pay. Thus, the fee agreement was unconscionable
because it was akin to a mortgagee lending to a mortgagor knowing that the mortgagor
was going to default and the foreclosure was inevitable. Lawyer must have known (a)
that H wasn't going to be able to pay and (b) that the value of the lien on the home was
worth more than the value of the services to be provided.
Thus, the fee arrangement likely constituted an ethical violation.
Violating Rights of Third Parties
30
Lawyers cannot violate the rights of third parties in the course of representing a client.
To the extent the lien violates W's rights and Lawyer knew of this, he likely acted
unethically. Furthermore, if Lawyer knew that H could not rightfully encumber the family
house, then Lawyer arguably breached his duty of competent and candid representation
by not informing H that he couldn't offer a lien on his house without W's consent.
ANSWER B TO QUESTION 2
1. Is Wendy's interest in the family home subject to damages recovered for
31
injuries to the pedestrian hit by Hal under California law?
The parties were married and live in California. Thus, their property rights as a couple,
specifically with regard to the property acquired during the marriage, are governed by
California community property law. Whether the house was community or separate
property can be determined by the source of the asset, whether any presumptions
apply, and the actions of the parties during the marriage.
Community Presumption
There is a community presumption regarding property acquired during the marriage that
it is community property. This would apply to the family home given, as the facts state,
it was acquired during the marriage. The presumption can be rebutted by a showing
that the house was not actually acquired during the marriage, it was acquired during the
marriage but with separate property funds, the house was a gift/devise/inheritance, or
the house was the rent/issue/profit derived from separate property.
Their house was purchased during the marriage so it was not a gift or devise. Although
it is possible that the house was purchased with separate property funds, there are no
facts to indicate this was the case. Because it was purchased during the marriage, and
there are no facts to rebut the presumption, the house is considered community
property.
Judgments Against Spouses
A tort judgment against a spouse will subject both the community property and the
separate property of the tortfeasor to the judgment. But once the community property is
divided, debt cannot be recovered from the spouse who received her half of the
community property from what she received under the divorce decree unless she was
the spouse that incurred the debt or the debt was assigned to her. Thus, for a judgment
against Hal for drinking and driving, the community will be liable for this debt, and it can
be satisfied from the community property.
For the Benefit of the Community
32
Although the community property is liable for the judgment by the pedestrian, the
judgment must be satisfied first from the separate property of the tortfeasor spouse if
the tort was not committed by conduct that was being performed for the benefit of the
community. For example, if Hal was on his way to drop the kids off at school or to pay
the mortgage on the house, this would be for the benefit of the community. In that case,
the judgment would be satisfied first from community property, and if there was any
deficiency, then from the separate property of the tortfeasor.
Here, Hal had been kicked out [of] the house for his drinking problem at the time of the
accident. Wendy had clearly communicated her disapproval for Hal's drinking. The
drinking, including drinking and driving, would actually harm, not benefit, the community.
Although we do not know where Hal was headed, he had already been kicked out of the
house and was, generally, involved in a drinking binge at the time. Therefore, his
actions were not to the benefit of the community and can be satisfied first from his
separate property assets.
But the facts state that his only asset, at the present time, is his interest in the family
home. Because it appears he has no separate property from which to satisfy the
judgment, the judgment will be satisfied from the community property home.
End of the Economic Community
The accident in which the pedestrian was hit occurred after Hal had been kicked out of
the house but before Wendy told Hal she wanted a divorce. As stated above, the
source of property or debt, whether it was incurred before, during or after the marriage,
can indicate whether it is community or separate debt. The pedestrian's claim is a form
of debt because, once rendered, the plaintiff can reduce it to a judgment and attach
liens to the tortfeasor's property. Thus, the question arises whether the economic
community ended when Wendy kicked Hal out of the house, because if so, the injury
and judgment would have occurred after the economic community ended and would be
the separate debt of Hal. In this case, the judgment could not be satisfied from
community property, including the house.
In California, end of the economic community occurs when there is physical separation
and an intent not to carry on the marital relationship anymore. If the parties maintain
the facade or marriage, although physically separated, the economic community will not
be considered to be at an end. The economic community will certainly result, if the
above elements are not satisfied, when the divorce decree is entered.
Here, Wendy kicked Hal out of the house one year ago. She did not say anything about
ending the marriage or never wanting to see him again. She did tell him he could not
return until he completed alcohol treatment. Thus, Hal being kicked out was not
indicative of an intent to permanently end the marriage relationship, it was indicative of
a temporary physical separation by Wendy for the limited purpose of motivating Hal to
get treatment and save the marriage. Thus the economic community would not have
ended simply when he left the house.
But, after having moved out and hitting the pedestrian while drinking, Wendy learned of
the accident and told Hal she wanted a divorce. At this point, both elements would be
met. Hal and Wendy would have been physically separated, and one spouse has
indicated an intention not to resume the marital relation by telling the other she wants a
divorce.
Because the economic community did not end until that time, when Wendy told Hal she
wanted a divorce, and the accident and/or the cause of action that is the basis for any
judgment accrued before that time, the judgment resulting would be a community debt
because it was essentially incurred before the end of the economic community.
33
Debt
34
Debt incurred before or during the marriage can be satisfied from the community or from
the tortfeasor's separate property. Debt incurred by a spouse for necessaries, including
medical care, can be satisfied from community property or the separate property of
either spouse, although indemnity may be available. Here, the debt is for tort judgment
and, as stated above, can be satisfied from either community property or separate
property of Hal, first from his separate property and then from the community property.
In California, for the purpose of debt for necessaries or medical services, end of the
economic community can only occur on divorce. Judgment may not be able to be
satisfied from Wendy's earnings if she kept them in a separate (versus joint) account
from which Hal had no right of withdrawal.
CONCLUSION--Because the debt was incurred before end of the economic community,
it is a community debt. Therefore, it can be satisfied from community property or
separate property of Hal. Because the tort that is the basis of the judgment was not
conducted for the benefit of the community, the judgment must be satisfied first from
Hal's separate property. But because Hal has no separate property, his only asset is
the house, it will be reduced to judgment and recovery sought from the asset that is the
community home, which as above is classified as community property. Wendy may be
able to seek indemnity.
2. Is Wendy's interest in the family home subject to payment of Hal's legal fees
under California law?
As stated above, the economic community ended when Wendy kicked Hal out of the
house and told him she wanted a divorce. Hal appears from the facts to have consulted
the lawyer after that time. Debt incurred after the end of the economic community will
belong to the debtor spouse.
Attorney Fees for Divorce Lawyer
Generally, a spouse may not unilaterally encumber community real property without a
joint action on behalf of both spouses. Additionally, the spouse may not separately
encumber her half interest in the property. The one exception to this rule is for the
spouse to satisfy attorney fees in the divorce proceeding between the spouses.
Here, because the lawyer is not representing Hal as a family attorney in his anticipated
divorce proceeding with Wendy, this rule would not apply. The lawyer fees incurred by
Hal after the economic community ended for the purpose of defending against the tort
suit could only be satisfied from Hal's separate property.
Division of Assets on Divorce
35
Generally, assets are divided pro rata at divorce, 50-50, no cashing out one spouse to
give the other an entire asset. The only general exceptions to this rule are: for a closely
held corporation whose shares are community assets where one spouse is the CEO
and division would destroy the business; a pension plan from which one spouse can
take a cashout instead of receiving payments from the pension so the spouse, who no
longer wish to have any connection can go their separate ways; or, for the family home
when selling it and dividing the proceeds will uproot the children and cause them harm.
While this is the family home, there appear to be no children and no reason not to apply
the binding pro rata division, 50-50, by sale of the house and splitting the assets.
This means that on divorce, the assets of the house will be split evenly between the
parties. Once the divorce decree is entered, the proceeds from the house that Hal
receives are going to be his separate property. Upon divorce, the legal fees of Hal's
lawyer can be paid by his share of the proceeds.
But the question asks whether the payment of Hal's legal fees will be satisfied from
Wendy’s interest in the home. Wendy has no interest in Hal's proceeds after divorce
from sale of the community property house, and thus the proceeds subject Hal's
interest, not hers, to liability.
CONCLUSION--because the attorney fee debt will have been incurred after end of the
economic community, it will be separate debt of Hal, and does not subject any of
Wendy's interest in the family home to liability for those fees. The exception for divorce
attorney fees does not apply.
3. What ethical violations has the lawyer committed according to both the ABA
36
and California law?
A lawyer is a fiduciary of the client. She has a duty of confidentiality (not to
communicate information relating to representation), a duty of loyalty not to act on
behalf of her own, a client's, or a third party's best interests that are adverse to her
client's, financial duties, and duties of competence which are all owed to the client.
Duty of Loyalty
Under the duty of loyalty, the lawyer must not develop an interest or maintain an interest
that is adverse to the client, whether it is the interest of the lawyer herself, an interest of
one of the lawyer's other clients, or an interest of a third party with whom the lawyer is
closely related.
Loyalty--Financial Assistance to Clients
Under the ABA rules, a lawyer is not permitted to lend the client money for the
representation, with the exception of forwarding costs of litigation to indigent clients and
forwarding costs associated with a contingent fee arrangement. Under the California
rules, the lawyer can lend the client any amount for any reason, as long as she does not
promise to satisfy the existing debts of the client in order to buy the client's business.
Therefore, from this perspective, the loan would be considered acceptable under the
California rules but unacceptable under the ABA rules. Under the ABA rules, once the
client becomes indebted to the attorney, the attorney's personal interest against the
client in collecting the money and receiving payment for the debt may conflict with his
duty to act for the sole benefit of the client. Under the California rules, because this is
not a promise to satisfy pre-existing debt for the prospective client, this is acceptable.
Loyalty--Transacting Business or Developing Adverse Interest to Client
37
Whenever the lawyer enters into business with the client, the terms must be fair, the
lawyer must disclose the terms (effect of the transaction) to the client in writing, allow for
an opportunity for the client to consult with independent counsel and probably should
suggest she do so if the lawyer's interest will be adverse to the client's in the litigation,
and obtain consent from the client in writing.
This loan would essentially be such a transaction. The facts do not indicate the above
elements are met. Additionally, there is a question whether it would be fair to encumber
a client's sole asset in order to receive payment. But the above rules that specifically
address lending a client money are going to govern whether the transaction is
permissible. Regardless, even though the loan is permissible under California law, the
attorney should ethically consider whether the terms of the loan are fair and suggest
receiving independent legal advice if the client wishes to fund the representation in this
manner.
Financial Duties
The reason the nature of the fee arrangement is important is to judge whether it is
permissible for the lawyer to charge the client in this way. Under the ABA, the fee must
be reasonable considering the experience of the lawyer, novelty of the case, difficulty of
legal issues, time and effort required, etc. In California, it simply must not be
unconscionable. The question is whether the lawyer has complied with the
requirements for charging a fee, and whether the amount is justified.
Contingent Fee
A lawyer can enter into either an hourly fee arrangement or a contingent fee
arrangement with a client, or potentially a flat fee arrangement. Under the ABA rules,
contingent fee arrangements (lawyer forwards fees and sometimes costs in order for a
stake in the recovery, if there happens to be one) are not available in criminal or
domestic cases. They must include the percentage of recovery taken, the costs
deducted from recovery, and whether they are deducted before or after. In California,
the agreement must also indicate that it is subject to negotiation with the lawyer and
what costs will not be covered by the contingent fee arrangement.
Under ABA rules, this may be a criminal case, but considering the question implies a
money judgment that could subject the house to liability, brought by a private party
pedestrian; using contingent fee arrangement in this case would be permissible. But
here, if the mortgage is being used as payment, and thus this is more likely to be
considered an hourly fee arrangement.
Hourly Fee
38
The agreement, under ABA rules, must disclose the rate at which the fee is charged,
the services it covers, and the respective duties of lawyer and client. In California, it
must also be in writing unless it is for less than $1,000, with a corporate client, routine
matter for regular client, or emergency renders this impossible.
CONCLUSION--There is nothing in the facts to indicate the lawyer has complied with
any of the above requirements regarding the fee arrangement. He made the offer to
encumber the property without explaining the calculation of the rate, providing a writing,
explaining what services it would cover, etc. Additionally, the case appears to be a
simple one, involving culpability for drunk driving. Depending on how much the house
was worth, a lien on the home could be unreasonable or unconscionable under either
California or ABA approach.
Duty of Competence
A lawyer has a duty of competence, to represent the client with the skill, knowledge,
thoroughness and preparation necessary to carry out the representation effectively.
As stated above, the home is community property. It cannot be encumbered unless
both spouses jointly enter into the transaction. The non-consenting spouse can recover
the house even from a BFP, and set aside the transaction, if she has not agreed to it.
There is a one year statute of limitations, but if the buyer knew the seller was married
and failed to seek consent from the other spouse, there is no statute of limitations.
Here, an attempt to encumber the community property house to satisfy the separate
debt of Hal would be a failure of competence on the part of the lawyer. A lawyer of
reasonable skill, knowledge, thoroughness and preparation would be aware of this and
would not attempt to encumber property to pay his debts knowing it was community
property not subject to this type of transaction without consent of Wendy. This would
ineffectively carry out the representation.
CONCLUSION--Under ABA rules only, the lawyer has breached his duty of loyalty to
the client by lending him money in regard to the transaction. Although, he may argue
he is permitted to do so because he is permitted to forward costs of litigation to indigent
clients and Hal is indigent because he is unemployed and has no assets but the house.
But because the house cannot be encumbered this way without the consent of Wendy,
and a lawyer of reasonable skill and knowledge would know this, the attempt to
encumber the house without Wendy's permission may also be a breach of duty of
competence, subjecting the lawyer to discipline, sanctions, and malpractice liability.
There is also a question of whether the amount of the fee is reasonable or
unconscionable in light of the nature of the litigation and employment of the lawyer.
39
Question 3
Vicky was killed on a rainy night. The prosecution charged Dean, a business rival, with
her murder. It alleged that, on the night in question, he hid in the bushes outside her
home and shot her when she returned from work.
At Dean’s trial in a California court, the prosecution called Whitney, Dean’s wife, to
testify. One week after the murder, Whitney had found out that Dean had been dating
another woman and had moved out, stating the marriage was over. Still angry, Whitney
was willing to testify against Dean. After Whitney was called to the stand, the court
took a recess. During the recess, Dean and Whitney reconciled. Whitney decided not
to testify against Dean. The trial recommenced and the prosecutor asked Whitney if
she saw anything on Dean’s shoes the night of the murder. When Whitney refused to
answer, the court threatened to hold her in contempt. Reluctantly, Whitney testified that
she saw mud on Dean’s shoes.
The prosecution then called Ella, Dean’s next-door neighbor. Ella testified that, on the
night Vicky was killed, she was standing by an open window in her kitchen, which was
about 20 feet from an open window in Dean’s kitchen. She also testified that she saw
Dean and Whitney and she heard Dean tell Whitney, “I just killed the gal who stole my
biggest account.” Dean and Whitney did not know that Ella overheard their
conversation.
Dean called Fred, a friend, to testify. Fred testified that, on the day after Vicky was
killed, he was having lunch in a coffee shop when he saw Hit, a well-known gangster,
conversing at the next table with another gangster, Gus. Fred testified that he heard
Gus ask Hit if he had “taken care of the assignment concerning Vicky,” and that Hit then
drew his index finger across his own throat.
Assuming all appropriate objections and motions were timely made, did the court
properly:
1. Allow the prosecution to call Whitney? Discuss.
2. Admit the testimony of:
(a) Whitney? Discuss.
(b) Ella? Discuss.
(c) Fred? Discuss.
Answer according to California law.
40
Answer A to Question 3
California Proposition 8: Truth in Evidence Rule
41
Under Proposition 8 in California, all non-privileged, relevant evidence is admissible in a
criminal prosecution brought in California unless it falls within one of the specified
exceptions to the rule. Evidence that is admissible under Proposition 8 is still subject to
CEC 352 balancing.
Here, as this case involves the prosecution charging Dean with murder, Proposition 8
will apply to admit any evidence that is relevant and is not excluded for CEC 352
balancing.
1. Allow the Prosecution to call Whitney
The first issue is whether the prosecution should be allowed to call Whitney. This
depends on whether Whitney ("W") can claim one of the spousal privileges: spousal
communications privilege or spousal testimonial privilege.
Spousal Communications Privilege
The spousal communications privilege protects all confidential communications between
spouses that are made in the course of an existing marriage and in reliance on the
intimacy of the marriage. This privilege belongs to both spouses and may be claimed
by either to prevent the other spouse from testifying. Moreover, the privilege exists
regardless of whether the marriage has ended in divorce, so long as the communication
itself was made during a period when the marriage existed. For purposes of the
privilege, marriage does not end until there is a valid divorce.
Here, Whitney was called by the prosecution to testify that she saw mud on Dean's
shoes. This observation occurred when Dean and W were still married as Dean and W
have yet to obtain a divorce and reconciled prior to W providing any testimony.
Although W and D had separated because W had discovered that D was dating another
woman and W had moved out, for the purpose of this privilege, it extends for any
communication made prior to divorce. Finally, as W was called to testify to an
observation, rather than a communication between W and Dean, it would not be
protected under the communications privilege.
Thus, this privilege would not apply to prevent W from testifying as she did or to prevent
her from taking the stand.
Spousal Testimonial Privilege
42
The spousal testimonial privilege allows one spouse to refuse to testify against another
spouse in any action. For this privilege to apply, a valid marriage must still exist. The
privilege belongs to the testifying spouse, as the privilege is designed to protect the
harmony of the marriage, which is not salvageable if the testifying spouse wishes to
testify. Moreover, in California, the privilege allows the testifying spouse to avoid taking
the stand entirely.
Here, W was called to the stand to testify that she saw mud on D's shoes during the
night of the murder. Although W and D had been separated, because W moved out and
stated the marriage was over when she discovered that D had been dating another
woman and moved out, the marriage had not ended for the purposes of the privilege,
which requires a valid divorce. As such, W was privileged to choose not to take the
stand.
In this case, W initially was angry and was willing to testify against D and thus agreed to
take the stand and testify. W actually took the stand and was sworn in, prior to the
recess in which W and D reconciled and W decided not to offer testimony. Thus, the
prosecution will argue that W waived the privilege because she took the stand and was
sworn under oath.
By contrast, W will assert that she did not waive the privilege because, although she
took the stand, she asserted the privilege the first time that she was asked a question
by the prosecution. W refused to answer when court resumed and the prosecutor
asked W if she saw anything on D's shoes at the night of the murder.
As W asserted the privilege prior to answering any questions, the court will find that she
had a spousal testimonial privilege and could not be forced to testify against D.
However, W took the stand voluntarily and thus it was proper to allow the prosecution to
call W because she was the holder of the privilege and had not yet claimed it.
Proposition 8 does not allow privileged information to be admitted and thus will not
change the outcome.
2. Admit the Testimony
43
(a) Whitney
The first issue is whether the court should have admitted the testimony of Whitney.
Logical Relevance
Under California law, evidence is relevant if it makes a fact of consequence that is
actually in dispute more or less probable then it would be without the evidence.
Here, W testified that she saw mud on D's shoes. As V was killed on a rainy night, and
the prosecution was arguing that D hid in the bushes outside her home and shot her
when she returned from work, this evidence would make it more likely that D was
present in a muddy flowerbed and committed the murder.
Thus, it is relevant.
Legal Relevance
Evidence is legally relevant if its probative value is not substantially outweighed by the
danger of unfair prejudice, confusion of the issues, misleading the jury, waste, or undue
delay.
Here, D will argue that the testimony about mud on his shoes is likely to confuse and
mislead the jury, particularly if the prosecution has failed to establish that the mud came
from a flowerbed near Vicky's home. However, as this evidence has high probative
value in that it shows that D was standing outside in mud on a rainy night, it will likely be
admitted. Thus, this objection will fail.
Personal Knowledge
44
In order to be competent to testify, a witness must have personal knowledge of the facts
to which she is testifying based upon her percipient observations.
Here, W saw mud on D's shoes in the night in question and thus testimony about the
state of the shoes is within her perception and personal knowledge.
Spousal Communications Privilege
As discussed above, this will not protect W's testimony about the mud on D's shoes as it
was not a communication, but was an observation.
Spousal Testimonial Privilege
As discussed above, this will protect W's testimony because she is still married to D and
therefore cannot be compelled to offer evidence against him in the criminal action. Prop
8 does not change the outcome as privileged information is excluded.
Conclusion
W's testimony will be excluded as a result of the spousal testimonial privilege.
(b) Ella
The second issue is the admissibility of Ella's testimony.
Logical Relevance
See rule above.
Ella's testimony that she overheard D tell W that he "just killed the gal who stole my
biggest account" is highly relevant to the case. D is charged with murder and his
alleged motivation for killing Vicky is that they were business rivals. The statement thus
indicates that D committed V's murder, particularly because it was made on the night
that V was killed. This fact is in dispute as it relates to whether or not D is guilty of the
crime with which he is charged. Thus, this testimony is logically relevant.
Legal Relevance
45
See rule above.
Although D will argue that this statement is highly prejudicial and should be excluded
because it could be misinterpreted and it fails to identify V specifically, the court will
likely find that its probative value in showing that D committed the murder and that he
had a motivation to commit the murder far outweighs the risk of prejudice. Moreover,
the information goes to the heart of D's guilt or innocence.
Thus, the evidence will not be excluded on this ground.
Personal Knowledge
See rule above.
Here, Ella was standing by an open window in her kitchen, which was about 20 feet
from an open window in D's kitchen. Ella could both see D and W and could hear D tell
W that "I just killed the gal who stole my biggest account." Thus, Ella's testimony was
based on her percipient observations as she could personally see and hear what was
happening in D and W's house.
Thus, this objection will be overruled.
Hearsay
Hearsay is an out-of-court statement that is offered to prove the truth of the matter
asserted. Hearsay is inadmissible unless it falls within an exception or is being used for
a non-hearsay purpose. Proposition 8 will not apply to admit otherwise inadmissible
hearsay as hearsay is an exception to Proposition 8.
Here, Ella's testimony that D told W, "I just killed the gal who stole my biggest account"
is offered to show that D was in fact the person who killed V. Thus, it is an out-of-court
statement offered to prove the truth of the matter asserted and is only admissible if it
falls within an exception.
Party-Opponent Admission
46
A statement by a party-opponent regarding a relevant fact of the case is admissible over
a hearsay objection as it is a California exception from the hearsay prohibition.
Here, the statement that Ella testified about was a statement by D, who is the defendant
in the criminal action. This statement is highly relevant to the issues involved in the
case because it indicates whether or not D actually committed a murder of V, for which
he is being charged.
Thus, this exception would allow the statement to be admitted.
Statement Against Interest
A statement is admissible under an exception if it qualifies as a statement against
interest. A statement against interest is a statement of a now unavailable witness that
was against the person's proprietary, pecuniary, penal, or social interest when made
and that the declarant knew was against his interest when made.
Here, D made the statement to W that "I just killed the gal who stole my biggest
account." This statement would be against D's penal interest, because it could subject
him to prosecution for murder. Moreover, it could subject him to social ridicule,
ostracism and humiliation because he would be labeled as a murderer. D will argue
that the statement was not against his interest because it was made to his spouse in
reliance on the confidentiality of their marital relationship and thus he did not think that it
could be used against him. Moreover, he did not believe at the time it was made that it
would subject him to social disgrace as he expected his spouse to maintain the
confidentiality of the statement. As D likely did not know that the statement could be
used against his interest when it was made, this exception likely would not apply.
A declarant is unavailable if he can claim a privilege against testifying. As D can claim
the privilege against self-incrimination under the Fifth Amendment, he would be
considered unavailable for the purposes of this exception.
Thus, this exception would not apply because D likely did not know it was against his
interest when made.
Spontaneous Statement
47
A spontaneous statement is a statement made shortly after witnessing a startling event
and while the declarant was still under the stress of excitement.
Here, D made his statement to W and said "I just killed the gal..." indicating that he may
still have been under the stress of excitement from the murder. Moreover, a murder is
likely a startling event, especially when it involved hiding in the bushes and shooting
someone at their home and then seeking to avoid detection.
Thus, D's statement might be a spontaneous statement if he was still experiencing the
stress of excitement.
Contemporaneous Statement
A contemporaneous statement is a statement made at or near the time of an event that
explains or describes the defendant's actions.
Here, D told W, "I just killed the gal who stole my biggest account." Because D
specified that he "just" killed a gal, the statement may have been made near the time of
the event. Moreover, the statement describes D's own conduct in killing the gal and
explains his reasons for that conduct--she "stole my biggest account."
Therefore, provided it was made sufficiently close in time, it may qualify as a
contemporaneous statement.
Spousal Communications Privilege
48
See rule above. In addition, the spousal communications privilege is waived if the
privilege is not made in reliance on the intimacy of the marriage. A statement is not
made in this reliance, if it is made in the presence of a third person who does not fall
within the privilege. If the spouses could not have reasonably foreseen that the
communication would be overheard by a third party, then the privilege is not waived and
D may prevent Ella from testifying on the basis of the privilege. However, if the spouses
made the statement negligently when it could be overheard by a third party, then the
privilege has been waived as no reasonable efforts were made to maintain its
confidentiality.
Here, D and W had a conversation in their kitchen. No one else was present in the
home and D and W were having an intimate conversation as spouses, thus suggesting
that the conversation was made in reliance on the intimacy of the marriage. However,
D and W had this conversation while the window to their kitchen was open. This
window was only 20 feet from a neighbor's window which was also open and D was
talking in a sufficiently loud voice such that Ella could overhear the conversation. But,
because D and W engaged in a private communication between themselves and they
did not know that Ella overheard the communication, they likely were not so negligent
as to waive the confidentiality of the communications. D and W could rely on the
privacy of their home, even with an open window.
Thus, the spousal communication privilege will prevent this testimony.
(c) Fred
Logical Relevance
49
Fred's testimony that the day after Vicky was killed he was having lunch and heard that
two gangsters had "taken care of the assignment concerning Vicky" is relevant to
establish that Dean was not the person who killed Vicky. As whether or not D killed
Vicky is the primary issue in the murder trial, this is both highly relevant and in dispute.
This objection will be overruled.
Personal Knowledge
Here, Fred was having lunch at a coffee shop when he saw Hit and Gus conversing and
overheard the conversation. Thus, Fred had personal knowledge regarding the
statements that were made.
This objection will be overruled.
Hearsay
See rule above.
Here, F is offering testimony regarding the statements of both H and G, and both of
these statements must fall within a hearsay exception in order to be admitted. These
statements are offered to show that F and G committed the murder of Vicky.
G's Statement
Effect on Hearer
D will argue that G's statement asking whether H had "taken care of the assignment
concerning Vicky" is not offered to show the truth of that statement, as it was a question,
but instead to show its effect on H, who answered the question.
A statement offered to show the effect on the hearer is not hearsay and is admissible
over a hearsay objection.
Here, as this question is offered to show the effect on H in answering, it will be
admissible.
H's Statement
50
Although H merely made a gesture by drawing an index finger across his throat, such
an action can qualify as hearsay if it is intended to communicate.
Here, H's conduct was done in order to answer G's question regarding whether or not H
had "taken care of the assignment concerning Vicky." As this was intended to
communicate that H had in fact gotten rid of Vicky, it will qualify as hearsay.
Statement Against Interest
Here, this statement is against H's penal interest as he would be subject to prosecution
for murder if he killed Vicky. As H made this statement while at a coffee shop where
other people like F were around, H would know that he could be subject to punishment
for making it at the time it was made. It is unclear whether H is unavailable and the
admissibility will depend on this.
Thus, this is likely admissible testimony.
ANSWER B TO QUESTION 3
People v. Dean
51
1. Did the court properly allow the prosecution to call Whitney?
Spousal Testimonial Privilege
The California Evidence Code (CEC) contains a spousal privilege. The spousal
privilege allows a defendant's spouse to refuse to take the witness stand and testify
against his or her spouse. Although Dean's trial is a criminal trial, the CEC makes no
distinction between criminal and civil trials--the spouse may refuse to testify against his
or her spouse in either civil or criminal trials.
The spouse and defendant must be married during the time of trial. Here,
although Whitney had moved out of the house prior to Dean's trial and said the
"marriage was over," there is nothing to indicate that Whitney and Dean's marriage was
legally dissolved. Thus, Whitney was married to Dean at the time of trial, and therefore
can invoke the spousal testimonial privilege.
The spouse--not the defendant--is the holder of the privilege. Thus, even if Dean
did not want Whitney to testify against him, Whitney could if she so chose, and so long
as the matter she testified to was not otherwise privileged.
Under the CEC, the witness spouse may refuse to take the witness stand
completely. Here, although Whitney initially took the stand, intending to testify against
Dean, she could have refused to take the stand altogether. The issue is whether
Whitney could later invoke the privilege after voluntarily waiving the spousal testimonial
privilege.
The CEC does not dictate that a spouse has waived the spousal testimonial
privilege once he or she takes the witness stand. Here, Whitney has testified to nothing
yet. Thus, although she has taken the witness stand, she is still not otherwise
prohibited from invoking the spousal testimonial privilege. Thus, her testimony should
not have been compelled.
However, the court did not err in allowing the prosecution to call Whitney to the
witness stand because Whitney initially wanted to testify against Dean. Thus, error, if
any, was on the court's compelling Whitney to testify, not on the court allowing the
prosecution to call Whitney to the witness stand.
2. Did the court properly admit the testimony of Whitney, Ella, and Fred?
52
Whitney
Logical Relevance
To be admissible, evidence must be relevant. Under the CEC, evidence is
relevant if it has any tendency to make the existence of some fact of consequence to
the action more or less probable than the absence of such evidence. The CEC further
requires that to be relevant, the fact must be in dispute.
Here, Whitney's testimony that she saw mud on Dean's shoes is relevant
because it makes a disputed fact--whether Dean was hiding in the bushes outside
Vicky's home that rainy night--more probable than the absence of the evidence.
Legal Relevance
Even if logically relevant, the court may exclude evidence if its probative value is
substantially outweighed by the risk of unfair prejudice, confusing the issues, or
misleading the jury. Here, the probative value of Whitney's testimony is relatively high.
Because Whitney is Dean's wife, her testimony tending to inculpate Dean is especially
probative. That Dean had mud on his shoes the night of the murder tends to show that
Dean might have been hiding in the bushes that night. There is little risk of unfair
prejudice because there is nothing to indicate that Whitney's testimony that she saw
mud on Dean's shoes will cause the jury to have prejudice against Dean.
Spousal Testimonial Privilege
53
As discussed above, Whitney should have been able to invoke the spousal
testimonial privilege because she is married to Dean at the time of trial and thus may
refuse to testify against him. Although she took the stand--which California allows a
spouse to refuse to do--Whitney still had the privilege to not testify against Dean.
Confidential Marital Communications Privilege
Whitney may attempt to alternatively invoke the confidential marital
communications privilege. Any confidential communication between spouses is
privileged and inadmissible. Here, however, Whitney testified as to an observation, not
a communication. Whitney merely saw mud on Dean's shoes. Whitney did not testify
as to any communication Dean made to her. Thus, the confidential marital
communications privilege does not apply.
In conclusion, Whitney's testimony--although relevant--should have been
excluded because of the spousal testimonial privilege.
Ella
Logical and Legal Relevance
Ella's testimony that Dean told Whitney "I just killed the gal who stole my biggest
account" is extremely relevant. If Dean told Whitney this, it tends to make it more
probable that Dean in fact did kill Vicky. The probative value is high, and there is little
risk of unfair prejudice as a result of Dean's statement to Whitney.
Hearsay
Ella's testimony may be objected to on the grounds that it is hearsay. Hearsay is
an out of court statement being offered to prove the truth of the matter contained
therein. Here, Dean's statement is out of court because it was made in his home to his
wife. If offered to prove that Dean did kill Vicky, it would be being offered for its truth.
Thus, the statement is hearsay by definition.
Nonhearsay: Declarant's state of mind
54
Dean's statement may be offered for the nonhearsay purpose of showing his
state of mind. It could be offered to show Dean's intent to kill, rather than the fact that
he did kill Vicky. However, if offered only for this purpose, it would be highly prejudicial
because it would be very difficult for a jury to not consider the statement as evidence
that Dean actually killed Vicky. Thus, it should not likely be admissible solely for this
purpose.
Admission of a party/opponent
Alternatively, Dean's statement to Whitney could be offered for its truth if it comes
under a hearsay exception. The CEC provides an exception to the hearsay rule for
admissions made by parties and offered by an opponent. Here, Dean's statement to
Whitney is a statement made by Dean--a party--and offered by the prosecution--an
opponent. Thus, although hearsay, Dean's statement may be admissible as an
admission--an exception to the CEC's rule against hearsay.
Confidential Marital Communications
However, Dean may seek to exclude his statement to Whitney on the grounds
that the statement was a confidential communication between spouses and thus is
privileged. Both spouses are holders of the privilege. Here there is a twist because a
third person is attempting to testify as to a confidential communication between
spouses. Both Dean and Whitney did not know that Ella overheard their conversation.
Thus, Dean and Whitney believed Dean's statement to be in confidence. Ella was
standing 20 feet away and in the house next door when the statement was made. If
Dean and Whitney's belief that the communication was confidential was reasonable,
such communication was privileged. Here, it appears that Dean and Whitney's belief
that their communication was in confidence was reasonable--notwithstanding the fact
that Ella overheard the communication 20 feet away.
The purpose of the confidential marital communications privilege is to foster the
confidence of the marital relationship, and to encourage open and honest
communication. Here, if Ella is permitted to testify as to Dean's statement if Dean and
Whitney reasonably believed their communication was made in confidence, such an
allowance would seem to go against the grain of the purpose of the confidential marital
communications privilege. Spouses should not have to take every measure to ensure
their communications are confidential so as to invoke the benefit of the confidential
marital communications privilege. A reasonable belief that the communication is made
in confidence should be sufficient. Here, the court should not allow Ella's testimony for
this reason.
Logical and Legal Relevance
55
Fred's testimony that Hit implicitly admitted to killing Vicky is relevant because it
makes it more probable that Dean did not kill Vicky. Assuming that the Vicky that Gus
was talking about was the same Vicky who died the day before, such evidence would
be extremely probative to show that Dean was not the killer, but Hit was.
Hearsay
Hearsay is an out-of-court statement. To be a statement, there must be some
assertive words or conduct. Although Gus's question to hit was out of court, it was not a
statement because it was not assertive. A question is not an assertion. Thus, Gus's
question to Hit whether Hit had taken care of the assignment concerning Vicky was not
hearsay.
The issue becomes whether Hit's drawing his index finger across his throat was
assertive conduct. Taken in light of the surrounding circumstances, Hit's conduct
seems to indicate that Hit acknowledged to Gus that he in fact killed Vicky. To be
hearsay, the declarant need not utter actual words. Here, the judge would use his or
her discretion in deciding whether Hit's conduct was assertive. The court should hold
that the conduct was assertive when taken in context with Gus's immediately preceding
question.
Because Hit's assertive conduct was made out of court, and if offered to prove
the truth--that Hit did kill Vicky--it is hearsay by definition. Hearsay is inadmissible
absent any exception.
Statement against Interest
56
Dean may argue that Hit's statement was a statement against interest. However,
for a statement against interest to be admissible, it must be shown that the declarant is
"unavailable" to testify. No such showing has been made, and therefore Hit's statement
may not be admitted as a statement against interest.
Admission
Hit's statement cannot come in as an admission because Hit is not a party to the
action.
Present Sense Impression/Contemporaneous Statement
Hit's statement may not be admitted under the present sense
impression/contemporaneous statement exception because Hit's statement was not
made either while killing Vicky or immediately thereafter. Also, Hit was not describing
his conduct, he merely made a motion tending to indicate that he killed Vicky. Thus,
this exception does not apply.
Confrontation Clause
The Sixth Amendment right to confrontation applies to the states, including
California, and provides that criminal defendants shall have the right to be confronted
with the witnesses against them. Here, because Dean is offering the out-of-court
statement made by Hit, the Sixth Amendment right of confrontation does not apply.
Conclusion
Because Hit's conduct was assertive, given the surrounding circumstances, and
because it is only relevant to prove the truth of his statement--that he killed Vicky, and
thus inferentially, Dean did not kill Vicky--Hit's statement was hearsay. Because no
exception to the rule against hearsay applies, Hit's statement should not have been
admitted.
57
Question 4
Peter responded to an advertisement placed by Della, a dentist, seeking a dental
hygienist. After an interview, Della offered Peter the job and said she would either: (1)
pay him $50,000 per year; or (2) pay him $40,000 per year and agree to convey to him
a parcel of land, worth about $50,000, if he would agree to work for her for three
consecutive years. Peter accepted the offer and said, “I’d like to go with the second
option, but I would like a commitment for an additional three years after the first three.”
Della said, “Good, I’d like you to start next week.”
After Peter started work, Della handed him a letter she had signed which stated only
that he had agreed to work as a dental hygienist at a salary of $40,000 per year.
After Peter had worked for two years and nine months, Della decided that she would
sell the parcel of land and not convey it to him. Even though she had always been
satisfied with his work, she fired him.
What rights does Peter have and what remedies might he obtain as to employment and
the parcel of land? Discuss.
58
ANSWER A TO QUESTION 4
What rights does Peter have?
The first issue is what law should apply. The UCC applies if the contract is for sale of
goods. The common law applies if in all other circumstances, including a contract for
services or land. In this case, there is an employment contract that contemplates the
payment of a salary and a land conveyance in exchange for services. Thus, the
common law applies to this contract.
The second issue is whether there is a valid contract. A valid contract requires offer,
acceptance, and consideration. An offer exists if the offeror offers the offeree a deal and
signals that acceptance will conclude the deal. An acceptance occurs if the offeree
agrees to the terms of the offeror and gives the offeree notice of his assent.
Consideration exists if there is a bargained-for exchange and legal detriment (which
involves perform [SIC] in a way that one is not legally required to perform). Acceptance
only exists if the offeree consents to the exact terms of the offeror, also known as the
mirror image rule. If the offeree attempts to change any terms of the offer, then there is
an effective rejection and counteroffer. Della advertised for a dental hygienist.
Advertisements are not usually considered offers and Della's advertisement did not
indicate that anyone who responded would be hired. The need to conduct an interview
suggests that Della's advertisement was an invitation to make an offer, not an actually
offer. Della interviewed Peter and offered him a job. She gave him a choice of being
paid $50,000 per year, or being paid $40,000 per year and the conveyance of a $50,000
parcel of land at the completion of three years of work. This might have been an offer
because it signaled to Peter that the deal would be complete if he chose either option.
However, it would more likely be considered preliminary negotiations since Peter could
still choose which option he preferred. Peter said, "I'd like to go with the second
option..." If there was an offer, and he had left his statement at this, then this would
constitute acceptance because it gave Della notice that he was accepting her offer.
However, Peter attempted to modify the terms of the deal by adding a commitment for
59
an additional three years after the first three years. Thus, Peter's attempted acceptance
was ineffective because it altered the terms of Della's offer and does not meet the mirror
image rule. Rather, Peter effectively made a counteroffer to Della (or an offer if Della's
original options were considered preliminary negotiations). Della accepted Peter's
counteroffer when she said, "Good, I'd like you to start next week." The exchange of six
years of dental hygienist services for a $50,000 parcel of land and a $40,000 per year
salary constitutes consideration. Because there was an offer, an acceptance, and
consideration, there is a valid contract.
The third issue is whether the statute of frauds makes the service or land contract
unenforceable. The statute of frauds requires some contracts to be in a writing signed
by the party against whom enforcement is sought. Contracts for land and contracts that
cannot be completed within a year are both included within the statute of frauds.
Contracts for land must adequately identify the parties and the parcel of land to be
conveyed. The contract between Della and Peter was for six years of employment.
Peter could not complete his performance of six years of services within one year, thus
this contract falls within the statute of frauds. The contract between Della and Peter also
contemplated the conveyance of an interest in land. Della did sign a contract with
Peter, but the contract only specified that Peter agreed to work as a dental hygienist for
a salary of $40,000 per year. The conveyance of land was not considered within the
signed contract, nor was the length of the term of employment. Thus, the contract Della
signed cannot be used to overcome the statute of frauds. The employment contract for
a term of years and the land conveyance are both unenforceable under the statute of
frauds.
The fourth issue is whether Peter can overcome the Statute of Frauds defense via the
doctrine of part performance or equitable estoppel. Part performance in a land
conveyance requires that the party who seeks to enforce the contract must have
engaged in partial performance, which is usually evidenced by possession or payment
of the purchase price. Equitable estoppel requires that the party who seeks to enforce
the contract show that there was a promise and that the party reasonably relied upon
60
that promise to their detriment. It will probably be difficult for Peter to show partial
performance since he has not taken possession of the land or paid the full purchase
price. He might be able to argue that he has "paid" a substantial portion of the purchase
price since he worked for two years and nine months, which is the equivalent of 75% of
the service he was to perform before receiving the land. However, equitable estoppel is
probably a better argument for him to make. The fact that Della offered Peter two
options suggests that $40,000 was less than the market rate for dental hygienists. Peter
chose the option that gave him less yearly salary in reliance on Della's promise that he
would be employed for six years and would receive a $50,000 parcel of land. He
received less salary than he otherwise would have, so his reliance was detrimental.
Peter may be able to overcome Della's Statute of Frauds defense under the doctrine of
equitable estoppel.
The fifth issue is whether there was a breach of contract. A breach occurs when one
party fails to perform as obligated under the express and implied conditions in the
contract. Assuming that the court finds a valid and enforceable contract, then Della
committed a breach when she fired Peter before the six years were complete. She also
committed an anticipatory repudiation when she decided to sell the land instead of
convey the land to him. She also potentially breached her implied duty of good faith by
firing Peter when she was satisfied with his work.
What remedies might Peter obtain?
The first issue is whether Peter can receive expectation damages. The general measure
of damages in a contracts case attempts to put the plaintiff into the position he would
have been in if the contract had been fully performed. A plaintiff does have a duty to
mitigate, which requires that he make a reasonable effort to find similar employment. He
does not have to settle for lesser employment or move to a distant location to find
employment. Assuming that the court finds there was an employment contract for six
years, the court would award three years and three months worth of the $40,000 per
year salary if Peter cannot find similar employment. If Peter can find similar
61
employment, the reward will be reduced based on whatever his new salary is. Assuming
that the court finds there was a contract to convey land, Peter could sue for the value of
the land, which was $50,000. If the court finds that there was an employment contract,
but no contract to convey land, then Peter might be able to receive more than the
$40,000 per year salary award if he can show that he took a reduced salary in reliance
on the promise that he would receive a land conveyance.
The second issue is whether Peter can receive restitutionary damages. Restitutionary
damages are only awarded when a benefit has been bestowed and it would unjustly
enrich the other party if they are not required to pay for that benefit. A plaintiff cannot
receive restitutionary remedies if they receive expectation damages. Restitutionary
damages would probably not be Peter's best option. However, Peter might be able to
receive the difference between his salary and the market rate salary for a dental
hygienist if he can show that he took the lower salary in reliance on the promise to
receive land.
The third issue is whether Peter can receive specific performance. Specific performance
is awarded when there is a definite and certain contract, an inadequate legal remedy,
enforcement of specific performance is feasible for the court, and there is mutuality. The
party attempting to avoid specific performance can do so by raising various defenses,
such as laches or unclean hands. Assuming Peter overcomes the statute of fraud
objections, Peter will not be able to seek specific performance for the employment
contract. Attempting to enforce an employment contract, which is a contract for personal
services, is not feasible for the court. Personal service and employment contracts
require individuals to work together in a cooperative environment; it is not feasible for
the court to monitor the relationship between the parties. Peter probably will not be able
to seek specific performance for the land contract. There was a definite and certain
contract to convey a parcel of land worth $50,000, though there may be some issues
with this element if it is not clear which parcel of land Della intended to convey. Land is
considered unique, so a legal remedy of $50,000 would be inadequate. It would be
feasible for the court to enforce the specific performance. Under the common law
62
doctrine of mutuality, both parties must have been able to request specific performance.
In this case, Della could not have sought specific performance if Peter breached.
However, under the modern theory, the requirement for mutuality is met if one party can
sufficiently assure performance. The court would have to decide if the two years and
nine months was enough to constitute full performance, but this is only 75% of the total
performance required. Peter may be willing to work the remaining three months, but the
court cannot require him to do it. Thus, there is no mutuality and Peter cannot
successfully obtain specific performance.
63
ANSWER B TO QUESTION 4
What Rights Does Peter Have as to Employment and the Parcel of Land
64
I. The Contract, if Valid, Is Governed By Common Law
The issue is what law governs the contract, if valid, between Peter (P) and Della
(D). The UCC governs contracts involving the sale of goods. Contracts which are for
services or are land contracts are governed by the common law. Here, P and D are
contracting for employment and possibly land. This is a contract for services and land
and therefore the contract is governed by common law principles.
II. There is Likely a Valid Contract Between Peter and Della
The issue is whether Peter and Della actually entered into a valid contract. For a
contract to be valid, it must contain offer, acceptance, and consideration. An offer is an
outward manifestation by the offeror that creates the power of acceptance in the
offeree. An advertisement can be a valid offer is it is made to a particular person,
outlines the specific details of the offer, and presents the recipient of the advertisement
with instructions as to how acceptance can be made. Acceptance is an outward
manifestation by the offeree that he accepts the terms of the offeror. Acceptance must
mirror the terms of the offer. If acceptance does not mirror the terms of the offer or, in
itself, alters the terms of the offer, it is a counteroffer and effectively rejects the original
offer. However, a mere inquiry is not a counteroffer. Consideration is a bargained-for
legal detriment. (i.e., A works for B in exchange for a salary).
Here, P responded to an advertisement from D, a dentist, who was seeking a
dental hygienist. The advertisement was not a valid offer because there are no facts
that it was sent directly to P, there are no facts that it contained the details of any
potential employment contract, and there are not facts that it told P how he could
accept. However, when D interviewed P, she presented him with a valid offer to be her
hygienist for three years in exchange for either (1) working for $50,000 per year; or (2)
working for $40,000 per year and she would agree to convey to him a parcel of land,
worth about $50,000. When P accepted, he said "I'd like to go with the second option,
but I would like a commitment for an additional three years after the first three." This
acceptance by P does not mirror the terms of the offer by D and therefore acts as both a
rejection of the offer and a counteroffer. Della said, "Good, I'd like you to start next
week."
Peter will argue that Della's comment of "Good, I'd like you to start next week," is
her acceptance of his counteroffer. He will argue that the terms of the deal are that he
works for Della for 6 years at $40,000 per year and is conveyed the parcel of land after
the first three years. When P started to work and D handed him the letter stating only
that he had agreed to work as a hygienist for $40,000 per year, P will argue that this
letter is merely a documentation of the salary he is to receive and nothing more.
In conclusion, Peter's counteroffer is the controlling offer and D accepted it by
saying, "Good, I'd like you to start next week." The consideration is that Peter work for
6 years at $40,000 and will receive the parcel of land at the completion of the first three
years. The consideration is valid. There is likely a valid contract between P and D.
III. The Letter D Presented to P Is An Invalid Modification
65
The issue is whether the letter D presented to P is an invalid modification. Under
the Common Law, a modification to a contract must be supported by consideration.
The pre-existing duty rule prohibits the modification of any contractual duties which
have been agreed to absent consideration because the party is attempting to modify
something that he/she is currently obligated to do.
Here, D attempted to modify the existing when she presented P with a letter,
which she signed, documenting P would work as a dental hygienist for $40,000 and no
other elements of the deal between P and D were documented. There was no
consideration paid by D to P to enforce this modification and it is invalid.
In conclusion, the modification is invalid because D is obligated to have P work
for 6 years at $40,000 and convey a piece of land to him after 3 years of work. To
reduce her obligations to only paying him $40,000 per year without consideration is in
violation of the pre-existing duty rule.
IV. Della Can Assert the Defense of Statute of Frauds (SOF)
66
The issue is whether D can assert a SOF defense. The SOF requires that
certain contracts be in writing. The categories are contracts regarding marriage,
contracts which cannot be performed within one year, land sale contracts, executor
agreements, guarantees or suretyships, and contracts for the sale of goods for over
$500. A contract which cannot be performed within one year is determined at the time
of the contract execution and is measured by whether there is any possibility
performance can be completed within one year. The writing that will satisfy the SOF
must contain the essential terms of the contract and be signed by the party to be
charged.
Here, P’s contract is for 6 years, or, at the least, 3 years, and is clearly not
performable within one year. This contract is subject to the statute of frauds. The
parties did not sign a written contract for Ps services to D. Further, part of the deal is a
land conveyance which is also subject to the SOF. Neither of those terms were ever
written down and D can assert that the contract fails under the SOF. Peter will argue
that the letter D gave to him after he started working is a writing confirming their contract
because it says he gets paid $40,000 and it is signed by D. However, this is not the
same contract to which they agreed.
In conclusion, it is likely that D can assert a valid SOF defense because the `
contract was not in a writing which comports with the requirements of the SOF.
V. P Can Assert The Defense of Estoppel and Likely Partial Performance to the SOF
67
Requirements.
The issue is whether P can assert the defenses of estoppel or part performance
to the SOF requirements. As stated above certain writings are subject to the SOF.
There are four defenses to the enforcement of the SOF: (1) Partial or Full Performance,
(2) Estoppel, (3) Judicial Acknowledgement of Contract, and (4) Merchant's
Confirmation Memo. There has been no acknowledgement in a judicial proceeding and
the merchant's confirmatory memo is only for UCC contracts with a merchant, so neither
apply. However, Partial or Full Performance and Estoppel may apply.
Partial or Full Performance
A party may not comply with the requirements of the SOF if he partially or fully
performs his contract and the other party accepts the benefits of the performance.
Here, P worked for D for 2 years and 9 months. At the very least, D was under the
impression that P was going to be working for her for 3 years, even though the final
accepted offer was likely for 6. There are no facts which say she failed to pay him so
she very likely was performing her obligations under the contract. She was accepting
his benefit of being a hygienist in exchange for her payment. Therefore, under the
doctrine of part performance, P has a meritorious defense to the requirments of the
SOF.
Equitable Estoppel
A party may not comply with the requirements of the SOF if he can assert a
defense of estoppel. Equitable estoppel occurs when a party says or does something
that foreseeably creates action in another person, the other person relies on the party's
previous statement or action, and it would be unjustly prejudicial to the relying party.
Here, P has fully relied on Ds statement of acceptance to his counteroffer. He began
working for her and has been working for her for almost 3 years. D has reason to know
that he was working for her based on their discussions of the $40,000 and land
conveyance. P may not have started working for D without the provisions agreed to in
his counteroffer and therefore it would be unfairly prejudicial not to enforce his contract.
In conclusion, P has a likely defense of partial or full performance ot the SOF and
may have a mertitorious defense of Estoppel.
VI. If A Valid Contract Exists, It is A Contract For Term and Not an At-Will Contract
68
The issue is whether the contract is a contract for term or an at-will contract. In a
contract for term, an employee has a property right in the job and may not be terminated
without cause. Conversely, an at-will contract allows the employer or employee to
terminate employment for good cause, bad cause, or no cause.
Here, P will argue that this is a contract for terms because the terms of his
counteroffer were that he worked for D for 6 years. Further, he will argue that even if
her original job offer is controlling, that offer was for a 3 year term. Either way, it is not
an at-will employment. Since it was not at will, she was not able to fire him because she
had always been happy with his work. Della will argue that her letter modifying the
contract has no language regarding term and therefore it is an at-will employment and
she can fire him for any reason.
In conclusion, this is a contract for term and P may not be fired absent cause.
In conclusion, P and D have a valid contract for 6 years at $40,000 per year.
Further, D is obligated by the contract to convey P the parcel of land upon completion of
his 3rd year. Peter has a right to seek remedies for breach of contract.
What Remedies Can Peter Seek
VII. Peter May Seek Expectation Damages and Reliance Damages
The issue is whether Peter may seek expectation damages and reliance
damages for his contract with Della. Legal remedies are available if the plaintiff can
clearly estimate the damages incurred with specificity. Legal damages are in three
categories, expectation, reliance, and restitution. Expectation damages place the
plaintiff in the position he would have been in had the breaching party performed the
contract in full. Reliance damages place the plaintiff in the place he would have been
had the contract not existed. Restitution damages reimburse the plaintiff for any benefit
conferred on the defendant. A plaintiff always has the duty to mitigate damages and, in
the employment context, the duty to to find other employment. The plaintiff is not
required to find any job, but rather a job comparable to the job that has been taken. If a
plaintiff cannot find replacement employment, a good faith effort must take place to find
employment.
Here, P will argue that he should get his expectation, or benefit of the bargain
damages, from the contract including any incidental and consequential damages that
are reasonably foreseeable from Ds breach.. He can easily estimate them because he
was due 3 years and 3 months salary and the parcel of land. He had a right to those
damages because he was under a contract for which he was improperly fired. These
damages will place him in the position he would have been in had he not been fired and
the contract been performed. However, he has a duty to find alternative employment
and there are no facts which say he has looked for or obtained any further employment.
Also, there are no facts that say he has acted in bad faith which would negate the award
of damages. If and when he does, his salary from that employment can be applied
against his damages from D. There are no facts indicating any incidental and
consequential damages.
Also, if P spent any money in reliance on his contract with D, he may recover
those costs that are reasonable and foreseeable. Any money that he spent in reliance
on the contract with D is obtainable.
In conclusion, he can obtain expectation and reliance damages from D less his
duty to mitigate by finding other, comparable employment.
VIII. Peter May Seek Specific Performance of the Land Contract, But Not the Services
69
Contract
The issue is whether Peter can seek specific performance of the land contract.
Specific performance is available when the contract has definite and certain terms,
there is an inadequate legal remedy, the court can correctly adjudicate, there is
mutuality between the parties and there are no defenses. Inadequate legal remedy
applies when you are dealing with land or unique items. Mutuality has been relaxed
and no longer requires that the parties must each be able to get specific performance.
Just that the party is ready and willing to perform. Specific performance will not be
applied to a services contract because it is difficult to enforce and can abridge certain
constitutional provisions against servitude.
Here, the land at issue is unique and is a definite term of the contract. Money
damages will not suffice. Peter contracted and performed for the piece of land. The
judge can properly adjudicate the matter. However, Peter likely may not seek specific
performance of the services contract.
In conclusion, P may seek specific performance of the land contract but not the
services contract.
70
Question 5
In 2004, Mae, a widow, executed a valid will, intentionally leaving out her daughter, Dot,
and giving 50 per cent of her estate to her son, Sam, and 50 per cent to Church.
In 2008, after a serious disagreement with Sam, Mae announced that she was revoking
her will, and then tore it in half in the presence of both Sam and Dot.
In 2010, after repeated requests by Sam, Mae handwrote and signed a document
declaring that she was thereby reviving her will. She attached all of the torn pages of
the will to the document. At the time she signed the document, she was entirely
dependent on Sam for food and shelter and companionship, and had not been allowed
by Sam to see or speak to anyone for months. By this time, Church had gone out of
existence.
In 2011, Mae died. Her sole survivors are Dot and Sam.
What rights, if any, do Dot and Sam have in Mae’s estate? Discuss.
Answer according to California law
71
ANSWER A TO QUESTION 5
Sam's Rights
72
In 2004, Mae executed a valid will that left 50% of her estate to her son, Sam,
and 50% of her estate to Church.
Revocation of 2004 Will
A will can be revoked by physical act. This requires that the testator tear, cancel,
obliterate, or destroy the will with the contemporaneous intent to revoke it. Here, in
2008, Mae had a disagreement with Sam and announced that she was revoking her will
as she tore the will in half, in the presence of both Sam and Dot. Because she
announced that she was revoking the will, that shows that she had an intent to revoke it.
Additionally, she got into a fight with Sam prior to this, and Sam was to take 50% of her
estate under that will. That further evidences that she intended to revoke the will. She
tore the will in half, which is a sufficient physical act. Thus, her actions in 2008 are
sufficient to count as a revocation by physical act. At this point in 2008, because Mae
revoked her only will, she does not have a testamentary instrument.
Revival in 2010
Holograph
A holographic will is one that is signed by the testator and all of the material
terms are in the testator's handwriting. Material terms are the beneficiaries and the gifts.
In 2010, Mae handwrote and signed a document that stated she was reviving her will.
Although it is signed by Mae and in her handwriting, the material terms are not in her
handwriting because they are referenced. Thus, this will only be a valid holograph if the
2004 will can be incorporated into the 2010 handwritten note because the 2004 will
contains the material terms.
Incorporation of the 2004 Will
A document will be incorporated as part of the will if it was physically present at
the time the will was executed and there was a simultaneous intent that the document
be a part of the will. Here, it seems that the torn pieces of the 2004 will were physically
present when Mae wrote the holograph because there are no facts suggesting she had
to go anywhere to get it; rather the facts seem to suggest that she wrote the holograph
and attached the torn pages in one sitting. Thus, it can be presumed that the prior will
was physically present when she wrote the holograph.
Furthermore, Mae had intent to incorporate the prior will because she physically
attached the torn pages of the will to the holograph document. This is sufficient to prove
her intent to incorporate.
Because the prior will was physically present and was intended to be a part of
the holograph, it will be revived in accordance with Mae's intent.
Incorporation by Reference
A writing can be incorporated by reference into a will if (1) there is a writing, (2) it
existed at the time of the will's execution, (3) it is specifically referenced in the will, and
(4) the testator had the intent to incorporate the writing.
Here, the 2004 will was in writing because it was valid at the time it was
executed, so it must have been in writing to be valid. It existed at the time of the will's
execution because Mae still had the torn pages. It is irrelevant that at that time it was
not a valid testamentary document, so long as it physically existed. It was specifically
referenced within the 2010 will because she stated that she wanted to revive her will,
and she only had one prior will that had been revoked. Furthermore, she attached the
torn pages to the 2010 will, so it is evident that she is talking about the 2004 will.
Because the first three elements are satisfied, there is a presumption that Mae had the
intent to incorporate the 2004 will into the 2010 holograph.
73
Independently Significant Fact
A fact is independently significant if it would have existed regardless of the
testamentary document being executed. Here, the 2004 will would have existed
regardless of the 2010 holograph because it was written prior to the 2010 holograph.
Even if Mae had never written the 2010 will, the 2004 will would have existed,
regardless of the fact that she revoked it. The torn pieces still remained. Thus, the 2004
will is independently significant.
Validity of 2010 Will: Undue Influence
74
Dot, who takes nothing under the revived will, will argue that the 2010 will was
the product of undue influence, and is therefore invalid, leaving Mae without a
testamentary instrument. There are three types of undue influence recognized in
California: the prima facie case, case law undue influence, or statutory undue influence.
Prima Facie Case
Under the prima facie case, undue influence can be shown if the testator was
susceptible to undue influence, if there was an opportunity to influence her, if there was
action taken to cause undue influence, and there was an unnatural disposition of the
estate because of the undue influence.
Here, Dot will argue that Mae was susceptible to undue influence by Sam
because she was entirely dependent on Sam for food, shelter, and companionship.
Thus, she was susceptible to doing what Sam wanted her to do. Dot will argue that Sam
had the opportunity to influence Mae because she was so dependent on him, Mae felt
that if she did not do what he wanted, she would have been left without food, shelter, or
companionship. There was active participation by Sam because he had repeatedly
requested that Mae revive the 2004 [will] and would not allow Mae to see or speak to
anyone for months. Finally, Dot will argue that the gift in the 2004 will was unnatural
because it did not provide for her, Dot, Mae's own daughter. Sam will argue, on the
other hand, that the gift revived by the 2010 will was not unnatural because it was a will
that was validly executed in 2004. There was nothing unnatural about it in 2004, and
there is nothing unnatural about it now. Furthermore, Mae intentionally left Dot out of the
will in 2004, so it was not unnatural to be left out now. Finally, Sam will argue that Mae
was not susceptible to any undue influence by him; rather he was just taking care of his
aging mother.
Ultimately, the court will probably side with Sam, that there was not an unnatural
disposition of Mae's property in the 2010 instrument because it was merely the revival of
a valid gift that she had already devised, despite the fact that she later revoked it. Thus,
the will will not be found invalid because of prima facie undue influence.
Case Law Undue Influence
Under case law undue influence, a gift or a will is invalid if there was a
confidential relationship between the testator and the person accused of having undue
influence, if there was active participation by the person causing the undue influence,
and if there was an unnatural gift because of the undue influence. Here, there is a
confidential relationship between Sam and Mae because Sam is Mae's son and he is
solely responsible for taking care of her. Mae is entirely dependent on Sam, so there is
a confidential relationship.
See above for arguments regarding active participation by Sam and the fact that
the gift was not an unnatural disposition of property.
Because the revival of the 2004 will by the 2010 will was not an unnatural
disposition of property, discussed above, there will be no undue influence.
Statutory Undue Influence
75
Under the California Probate Code, undue influence is presumed if the drafter of
the will is also the beneficiary of the will. Here, Mae handwrote the 2010 holograph and
attached the torn pages to that will herself. Thus, no one else drafted the will. The fact
that she did so at the repeated requests of Sam does not change the fact that he did not
draft the will leaving a gift to himself. Even if he did, there is an exception to this general
rule that if the drafter is also a relative of the testator, there is not going to be a
presumption of undue influence. Thus, there is no statutory undue influence.
Disposition re: Sam
76
If the court finds that there is no undue influence, the court will dispose of Mae's
estate in accordance with the 2010 will, which incorporates the 2004 will. Under that
document, Sam is entitled to 50% of Mae's estate, and Church is entitled to the other
50%.
Church: Lapse of Gift
Church was no longer in existence in 2010, when Mae executed her will. Thus,
her gift of 50% of the estate will lapse because Church does not exist and is not there to
take its gift.
Anti-Lapse?
California has an anti-lapse statute, which allows for the issue of a kindred
beneficiary to take, despite the fact that he or she may have predeceased the testator.
Here, however, Church is not kindred, or blood-related, to Mae, nor does it leave issue
because it is an entity. Thus, anti-lapse will not apply to Church's gift of 50%.
Remaining 50%: Intestacy
Because the gift of 50% of Mae's estate to Church will lapse, the will does not
provide for the distribution of that property. Thus, the remaining 50% of Mae's estate will
pass through intestacy.
Mae was a widow when she died, so she did not leave a surviving spouse. She
was survived solely by Dot and Sam, her children. Under the rules of intestacy, if a
decedent dies without a will or without full disposition of property by a will, the property
will go to the surviving issue, per capita. Under California Probate Code section 240,
you go to the first generation with living issue and divide the estate equally among
bloodlines with someone living. Here, Sam and Dot are both living, and they are in the
first generation. Thus, they will each take 50% of the remaining estate - in other words,
they will get 25% of Mae's estate each.
Dot's Rights
77
Dot was intentionally left out of the 2004 will, which later was revoked and then
incorporated into the 2010 will. Thus, under Mae's will, Dot stands to take nothing (with
the exception of her 25% intestate share due to the lapse of Church's gift).
Pretermitted Child
Dot will argue that she is a pretermitted child. A pretermitted child is one that was
not born or known about at the time the testamentary instrument was executed.
Pretermitted children are entitled to their intestate share of the entire estate. Thus, if Dot
is pretermitted, she will be entitled to 50% of Mae's estate because Mae's estate would
be split 50/50 between her two children in intestacy.
Here, Dot is not a pretermitted child because she was alive in 2004 when Mae
executed the will. Furthermore, Mae intentionally left her out of the 2004 will and she
revived that will, with the intent that it go back into effect. Therefore, Dot will not be
construed as a pretermitted child.
Distribution of Mae's Estate
If Dot is able to persuade the court that there was undue influence by Sam, his
gift will be invalidated because of the undue influence. If Sam's gift is invalid and
Church's gift lapse, that would mean Mae's entire estate would be distributed through
intestacy. In this case, Dot and Sam, as the sole surviving children, would be entitled to
50% each.
However, as discussed above, the court is unlikely to find that undue influence
will invalidate Sam's gift because it was not unnatural. Therefore, Sam will still be
entitled to his 50% under the will. Because Church's gift lapsed, however, the remaining
50% will be distributed under intestacy, with 25% going to each Sam and Dot. Thus, the
most likely distribution of Mae's estate results with Sam taking 75% of the estate, and
Dot taking 25%.
78
ANSWER B TO QUESTION 5
2004 - Valid Will
79
The facts here indicate that Mae executed a valid will in 2004 in which she intentionally
omitted D, and split her estate 50/50 between S and the Church.
2008 - Revocation
A will can be revoked by physical act or subsequent testamentary documents. When
revoking by physical act, testator, or someone under testator's direction must burn, tear,
destroy, or cancel the will. The testator must have the intent to revoke at the same time.
Here, in 2008, after a disagreement with S, M announced that she was revoking her will,
thereby indicating an intent to revoke, and then she tore it in half, fulfilling the necessary
physical act to revoke. Because she tore the entire will in half, there is an indication that
she intended to revoke the entire will, not just a part of it.
As such, Mae effectively revoked her 2008 will.
2010 - Revival
A will can only be revived if it was revoked by a subsequent testamentary instrument,
which was then later revoked by physical act or another testamentary instrument.
Revival re-effectuates an earlier will. Here, Mae's 2004 will was revoked by physical act,
not by testamentary instrument, so it cannot be revived by a document. Had this will
been revoked by a later instrument, S could argue that the first will was revived because
his mother executed a holographic codicil that explicitly stated that she intended the
earlier will be back in effect, and it would have been effective as of the date of the
codicil.
However, a will revoked by physical act cannot be revived.
2010 - Holographic Will
S could argue that in 2010, his mother executed a holographic will. A valid holographic
will requires that all material terms of the will be in the testator's handwriting, and it be
signed by her. Here, Mae wrote that she was reviving her will and she signed the
document. He could argue that even though this was not a valid revival, as discussed
above, it was a new will because testamentary intent can be inferred from her statement
that she wished to revive the earlier will, and she had signed and handwritten this new
will. Therefore, Sam may be able to argue that this was a new, valid holographic will.
To establish the terms of the will, he could look to integration, and incorporation.
Integration
80
A writing that is present at the time of the execution of a will, and is intended to
be a part of that will, is deemed to have been integrated into the will and is probated. An
intent to make it a part of the will can be established by it being attached to the will.
Here, S could argue that even though the previous will had been revoked, the pieces of
it were attached to the holographic will that his mother executed, and therefore, it was
integrated into the new will and should be probated. There is no requirement that the
attached documents be valid on their own. Therefore, Sam may be successful in
arguing that his mother's former will was integrated into the holographic will.
Incorporation by reference
A writing, whether valid or not, can also be incorporated by reference if it is in
existence at the time of the execution of the will, it is identified in the will, and there is an
intent to incorporate it. Sam could again argue that if his mother's will was not
integrated, it was incorporated by reference because she states in the new will that she
is reviving her former will, which indicates that she intended to incorporate it, and it is
clearly referenced in the new will. He can also argue that even though it was in two
pieces, it was still in existence at the time of the execution of this will. Thus, it was
incorporated by reference.
Undue Influence
Courts are unwilling to probate wills or terms of a will that are procured by undue
influence. Undue influence is when the testator's freewill is overcome. There are two
types of undue influence that the court may find were at play when Mae wrote the
document attempting to revive her former will: prima facie undue influence and undue
influence based on case law.
Prima facie
81
To establish a prima facie case of undue influence, a party contesting the will,
which in this case could be D because she receives nothing under her mother's initial
will, would have to show her mother's susceptibility to be influenced, her brother's
opportunity to influence Mae, S's active participation in influence, and an unnatural
result.
Susceptibility
Mae must have been in a vulnerable position in which her freewill could have
been overcome. In this case, she was completely dependent on S for her basic
necessities in life, such as food, shelter and companionship. Therefore, she was very
likely susceptible to having her freewill overcome by Sam.
Opportunity
S must also have had the opportunity to overcome Mae's freewill. In this case,
Sam did not allow Mae to see or speak to anyone for months, and his mother
completely relied upon him. Therefore, because he was her only source of
companionship, he had the opportunity to influence her.
Active participation
S must have actively influenced his mother. Here, he made repeated requests to
her to revive her former will, and it was only after these repeated requests that she did
so. Therefore, he actively participated.
An unnatural disposition
Proving an unnatural disposition may be difficult for D because the original will
devised half of Mae's property to S and that's also what the new will would do.
Furthermore, if Mae died intestate, he would still receive half of her property because
she only left behind two issues. However, because it is clear that Mae intended to tear
up her old will, and that this second document was only the result of S's pressure on
her, it may be possible for find undue influence.
Case law
82
Under the case law method of proving undue influence, there has to be a special
relationship between the influencer and the testator, active participation and an
unnatural result. Here, the special relationship can be established through the familial
bond, as S was Mae's son, and she was completely dependent on him to take care of
her. See above for the other two elements.
As a result, if the court were to find that there was undue influence, it would likely refuse
to probate the second will because the entire thing was obtained by such an influence.
On the other hand, because the disposition wasn't entirely unnatural, it may not find
undue influence, in which case it would be a valid will that could be probated.
Gift to the Church
In order to obtain a gift under the will, one must be in existence at the time of testator's
death. The church here was no longer in existence when Mae died. Under California's
lapse provisions, the gift to the church would lapse and fall into the either the residuary
clause of the testator's will, and if there wasn't one, then it would pass under intestacy.
The gift cannot be saved under the antilapse provisions because only kindred who
leave behind issue can benefit from that provision.
As such, if there was a valid will, the gift to the church would lapse, and as there is no
residuary clause, it would pass under intestacy.
Dot's Rights
Omitted Child
Dot could claim that she was an omitted child because she was not provided for in any
of Mae's wills. However, to be an omitted child, all testamentary documents must have
been executed prior to the birth of the child. Here, the facts clearly indicate that D was
alive when Mae executed her will in 2004, and then also again in 2010 if that is deemed
to be a valid will, and thus she was not an omitted child. Furthermore, Mae intentionally
left D out.
Intestacy Share
83
D's intestacy share will depend on whether the holographic will by Mae is considered
valid or invalid.
If the will is valid, 50% of her estate would pass under the will to S. The other 50% that
was to go to the church would have lapsed, as would pass under intestate distribution
as there is no document governing the disposition of that property.
Under the default rules for intestate distribution, when there is no surviving spouse,
which there isn't here because Mae was a widow, distribution to issue is on a "per
capita" basis. Each of Mae's children would get an equal share of the intestate property.
As Mae has two children, and 50% of her estate is passing by intestacy, D would get
25% of the total estate.
If on the other hand, the will is invalid, then all of Mae's estate would pass by intestacy.
Just as above, the property would be distributed equally between her two children, and
D would therefore get 50% of the estate.
Sam's Rights
Sam's rights to distribution will depend on whether the will is deemed invalid because of
his undue influence or because it was not a proper holographic will.
If the will is valid, S is entitled to receive 50% of Mae's estate under the will. The other
50% that would not pass to the church because it is no longer in existence would pass
through intestacy because of a lack of a residuary clause. Under intestacy, as
discussed above for D, Sam would receive 50% of the property that passes in such a
manner, which would result in a 25% share of the total estate. Overall, if the will is
deemed valid, Sam would receive 75% of Mae's estate.
If the will is not valid, then all of Mae's property would pass under intestacy, and S
would receive half just the same as D above. Therefore, he would get 50% of Mae's
estate.
Overall
84
Overall, the rights of D and S depend on whether the court finds that Mae had a valid
will at the time of her death. If there was a valid will, S would receive 75% of his
mother's estate, and D would receive 25%. If there was no valid will, then each S and D
would receive a 50% share.
Question 6
Dan worked at a church. One day a woman came to the church, told Dan she wanted
to donate some property to the church, and handed him an old book and a handgun.
Dan had originally intended to deliver both the book and the gun to the church’s
administrators, but he changed his mind and delivered only the book. He put the gun
on the front seat of his car.
The next day, as he was driving, Dan was stopped by a police officer at a sobriety
checkpoint at which officers stopped all cars and asked their drivers to exit briefly before
going on their way. The police officer explained the procedure and asked, “Would you
please exit the vehicle?”
Believing he had no choice, Dan said, “Okay.”
After Dan got out of his car, the police officer observed the gun on the front seat and
asked Dan if he was the owner. Dan answered, “No. I stole the gun. But I was
planning to give it back.”
Dan is charged with theft and moves to suppress the gun and his statement to the
police officer under the Fourth Amendment to the United States Constitution and
Miranda v. Arizona
85
.
1. Is Dan likely to prevail on his motion? Discuss.
2. If Dan does not prevail on his motion, is he likely to be convicted at trial? Discuss.
ANSWER A TO QUESTION 6
1. Is Dan ("D") likely to prevail on his motion?
86
A. On Fourth Amendment Grounds. The Fourth Amendment protects the citizenry from
unreasonable searches and seizures by the government. Thus violations require
government action. They also require that the search or seizure be unreasonable,
something that may be an issue for D. A search is a violation of a reasonable
expectation of property; a seizure is an instance in which a person does not feel "free to
leave" based on governmental presence. Generally, for a search to be reasonable,
there must be a warrant. A warrant is granted by a neutral judge and must be based on
articulable facts shown in an affidavit and must be reasonable and particular in terms of
scope and time. In this case, there was no warrant to search D's car or to seize D.
Thus, the search and seizure is presumptively unreasonable, subject to certain
exceptions. One important exception is the checkpoint search; another such exception
is consent. As an initial matter, a person must have standing to challenge the search.
Because Dan was driving his own car, he will have standing.
i) The Checkpoint Search: Warrantless, even suspicionless, road checkpoints have
been upheld by the Supreme Court under certain circumstances. First, the search must
be supported by the justification of highway safety - including prevention of DUI, etc.
Second, the checkpoints must be administered in such a way that officer discretion is
very limited. This means that an officer must go through a protocol driven method of
stopping the cars - i.e., either every car, or one of every ten cars, etc. The officer may
not stop whatever car he subjectively thinks looks criminal. Third, the search must be
reasonable in scope - it must not exceed the degree necessary to check for whatever
the search is aimed at.
Here, it does appear that the checkpoint search is aimed at a valid justification - a
sobriety checkpoint. This has been expressly held as constitutional by the Supreme
Court. However, there are some other issues. For one, all cars are being stopped.
While this is not presumptively unreasonable, it will be an issue, as it basically allows a
policeman to stop and seize every single person driving down the expressway.
Secondly, the police required D to step out of his car. Under Supreme Court precedent,
police only have been allowed to stop people. If sobriety or another criminal violation
seem likely, then the people can be asked to exit their car. Because of the stopping of
every car, and the demand that the drivers exit the car, this may be found to be an
unreasonably long stop than what is necessary to meet the highway safety justification.
Conclusion: There is a chance that this checkpoint too far exceeds permissible protocol
based on Supreme Court precedent. However, it is a close call. I will consider this to
be a reasonable and permissible warrantless search, though the court may be
convinced otherwise.
ii) Consent to Search: A person may validly waive his right to be free from
unreasonable search and seizure by giving consent. Because it is likely that the stop
and seizure was permissible up until the time that D was removed from his car, his
consent to get out of the car would completely remove any potential objection to the
search and seizure. The question will be whether the consent was freely and voluntarily
given. Courts have found that when police attempt to search a person's house on the
basis of consent, they do not have to tell that person that he or she has the right to
refuse consent. This does not remove the "voluntary" aspect of consent. Here, Dan
subjectively thought that he had no choice, but he still consented to getting out of the
car. Assuming that the court would apply the consent rule used in home searches to a
car search, this consent should be found to be voluntarily given.
Conclusion: Thus, the search for the gun was likely reasonable based on consent,
regardless of whether or not it was legitimate based on checkpoint rules for the cops to
remove him from his car.
iii) The Plain-View Doctrine: It appears, either because the entire checkpoint process
was constitutional, or because D gave his consent to be moved from the car after a
constitutionally permissible checkpoint stop, that the stop and seizure was constitutional
87
at the time Dan got out of the car. Thus, the police were constitutionally on solid ground
when Dan was out of the car. The plain-view doctrine allows police who are legitimately
in a place and see something criminal in plain-view to use that plain-view finding in
court. The justification is that a person does not have a reasonable expectation of
privacy in something the person lets the public see. Here, the gun will qualify under the
plain-view doctrine. The police need not rely on any Terry type frisks of automobiles, or
the automobile exception, because they do not apply. The gun was in plain-view, and
to the extent that the officer "searched" the car by looking in the window, the plain-view
exception applies.
iv) CONCLUSION: The search and seizure was reasonable and the gun should be
admissible. The checkpoint rule may validate the entire process, but even if it doesn't
then the checkpoint rule was at least legitimate up until the time D was asked to exit the
car. Because he consented, there is no violation of the 4th amendment. The gun is
admissible based on the plain-view doctrine.
B. Will D prevail on 5th Amendment Miranda Grounds? The 5th Amendment protects
the right against self-incrimination. Miranda v. Arizona, a case based on this right, holds
that a person's statements made cannot be used against him in court if the Miranda
warning is not given. However, Miranda applies only to custodial interrogations, and not
when a person is not in custody or voluntarily offers information. Miranda warnings
include the right to remain silent, the right to counsel, the knowledge that counsel will be
provided to a person, and the knowledge that anything said while in custody may be
used against that person in court.
i) No Miranda Warnings were given. Here, the cops gave no warnings. Thus, D's
statement is protected if it was made during a custodial interrogation.
a. Custodial. Custodial situations are those in which a reasonable, innocent person
does not feel free to terminate the encounter and leave at will. Here, D was out of his
car being asked in the company of some police. It seems up to this point to have been
88
a pretty friendly encounter, with the cops not showing much force or intimidation. Still,
it's hard to say whether someone would reasonably feel at this point justified and correct
in telling the police that this interview has to stop, and that the person is just going to
drive away; especially before the sobriety check is performed. Thus, it's a close call.
However, as D is out of his car, speaking to police, and about to be subject to a sobriety
test, I would conclude that this is a custodial situation as a reasonable person would not
feel free to terminate the questioning and leave.
b. Interrogation: An interrogative question is one that is reasonably likely to elicit an
incriminating response. This is a pretty close call as well. On one hand, the officers
had no indication that the gun was criminally possessed, and thus a mere question
about it may not be enough to reasonably expect an incriminating response. On the
other hand, if the gun was criminally possessed, then a truthful response would be
incriminating. However, because the officer questioned D about the gun without any
suspicion at all of it being stolen, I would find this to be a non-interrogative question.
I.e., if they knew that there was a stolen gun around, and then they asked, that would be
more likely to be an incriminating response. Here, this just seems like the officers
inquiring about a gun in the car without any suspicion whatsoever. Thus, Dan's
statement should be admissible. It also appears that even if he had denied the
ownership of the gun, the bit about him admitting to the crime was completely
volunteered. I.e., the cops did not ask him whether he stole the gun. They asked him if
he owned it. Thus, D's answer could have been "No." Instead, and completely
unprompted, D volunteered that he stole the gun.
ii) CONCLUSION: This was likely a custodial situation. The situation probably not
interrogative, but it may have been. Even if it was not an interrogative scenario, D's
statement that "I stole the gun" was not in response to any questioning by the police,
and is voluntary and admissible. If it is found to be an custodial interrogative situation,
the only part of the statement that will be inadmissible will be the answer to the
policeman's question: "No."
89
2. Which theft crime will D be convicted of?
90
A. Theft crimes are specific intent crimes. This means that the thief must specifically
intend the proscribed conduct - i.e., the thief must have the mens rea to permanently
deprive the true owner of the object possession. Theft crimes include larceny
(trespassory taking and carrying away of the personal property of another with intent to
permanently deprive); larceny by false pretenses (larceny, plus getting actual title to the
property by intentional and legitimate fraud); larceny by trick (larceny, but obtaining
mere possession of the property by trick or deception); and embezzlement (the
fraudulent conversion of the personal property of another by one legally in possession
of that property).
B. No larceny crime lies: This will be an embezzlement, if it's anything. The reason is
because the larceny crimes all require an intent to steal the item at the moment of
possession. Here, Dan did not form the intent to keep the gun until he had already
been in legitimate and lawful possession - as a courier for the church, and holding it for
the church. The continuing trespass doctrine will not apply, because that applies to
scenarios where a person has borrowed something against the owner's intent, but
doesn't plan to steal it until later. That person is never in lawful possession. Because
Dan's specific intent mens rea was not formed at the moment of possession of the gun,
no larceny crime will lie.
C. Embezzlement: Embezzlement is:
i) Fraudulent: I.e., wrongful. Here, D was supposed to deliver the gun to the church,
but has kept the gun. Thus, he is in wrongful possession of the gun at the time the gun
was found on him.
ii) Conversion: This means the intent to permanently deprive the owner (Church) of
possession. This will be the major issue. Dan tells the cops he wanted to give the gun
back; further we have no indication that he ever meant to keep the gun forever - maybe
he just wanted to drive around with it for a little bit. Because this is a specific intent
crime, the prosecution will have a tough job proving that Dan subjectively and
specifically intended to keep the gun forever when he decided to not turn it in. It is
important to note that once he kept the gun with intent to steal it, the crime was
complete - it doesn't matter if he later developed the intent to return it. The prosecution
could point to the fact that he was driving around with it and didn't turn it in when he was
supposed to, which may help; so will the statement that "I stole it." This will be the issue
at trial, right now it looks only probably proven at best.
iii) Of the personal property of another: The woman gave the gun to the church. As
such, the gun was the property of the church.
iv) By someone in legal possession: Dan worked for the church, and it was his job in
this instance to deliver the gun to the church. Thus, he has legal possession of the gun
when the woman gave it to him. She gave it to him thinking he was going to give it to
the church, because he was an employee of the church. The church charged him with
the duty of taking donations and delivering them to it. Thus, this possession was legal.
It is akin to a bank manager stealing money that he or she is supposed to be counting.
D. CONCLUSION: Embezzlement may lie, but only if the prosecution can prove
specific intent to steal the gun, which will be tough.
3. General conclusion: Gun and statement ("I stole it.") admissible.
91
Embezzlement if there is specific intent, which there likely is.
ANSWER B TO QUESTION 6
1. Motion to suppress
92
The fourth amendment prohibits unreasonable searches and seizures by the
state. Miranda v. Arizona requires that warnings be given to an individual subject to
"custodial interrogation" in order to protect the individual's right to be protected from
self-incrimination. This is clearly state action, so the issues here are whether the gun
was seized pursuant to an unreasonable search or seizure, or whether the statement
was obtained in the context of custodial interrogation.
Exclusionary Rule and Fruit of the poisonous tree doctrine
The exclusionary rule requires that a court exclude evidence seized pursuant to
an unlawful search or seizure. The fruit of the poisonous tree doctrine also provides
that evidence that is obtained as a result of an lawful search must also be excluded,
subject to certain exceptions. The exclusionary rule also requires the suppression of
statements obtained in violation of Miranda, although the fruit of the poisonous tree
doctrine does not apply to Miranda. Here, if the gun was seized during an unlawful
search or seizure, or if the statement was obtained in violation of Miranda, this evidence
must be suppressed.
Gun
Expectation of privacy
An individual has standing to challenge a search or seizure when they have a
reasonable expectation of privacy in the place or property being searched. When an
individual knowingly exposes something to the public, he no longer has standing to
challenge a search of it. In this case, Dan placed the gun on the front seat of his car. It
is not clear if his windows were tinted, or if someone could see easily into the car and
see the gun. However, typically an individual has an expectation of privacy as to the
inside and contents of their car, so Dan probably has standing to challenge the search.
He certainly has standing to challenge any detention of his person, which would
constitute a seizure if a reasonable person would not feel free to leave.
Routine checkpoint
93
Routine sobriety checkpoints are not considered seizures under the 4th
amendment, so long as they are administered in a nondiscretionary manner and do not
detain individuals for an unreasonable period of time. In this case, the officers at the
checkpoint were stopping all cars, and were asking all drivers to briefly exit before going
on their way. As a result, this checkpoint was not a seizure of Dan or his car, and did
not implicate the 4th amendment.
Consent
In addition, a search or seizure is not unreasonable if an individual consents to
the search. Valid consent must be knowingly and voluntarily given. Whether an
individual validly consented is determined objectively, and the court considers whether a
reasonable police officer would believe that the individual consented to the search or
seizure. In this case, the police officer explained the procedure and asked if Dan would
exit the vehicle. As a result, Dan appears to be informed about the procedure and his
consent was knowing. His consent was also voluntary because he said okay, and
stepped out of the car. A reasonable police officer would consider this to be valid
consent.
Plain-View
The plain-view doctrine provides that where a police officer has a right to be in
the place that he is, any objects in plain view may be validly searched or seized if there
is probable cause to believe that the objects are products or instrumentalities of a crime.
In this case, the officer had the right to be in the place that he was, as discussed above,
because he had the right to stop Dan pursuant to the nature of the checkpoint and
Dan's consent. At this time, the gun was in plain-view. The officer then asked Dan if
the gun was his, and he responded that it was stolen. At that time, the police officer had
not yet searched or seized the gun because he had not touched it or moved it in any
way. However, when Dan confessed that it was stolen, probable cause arose for the
officer to seize it, and the seizure was therefore lawful under the plain view doctrine.
Even if the statements were elicited in the context of a Miranda violation (to be
discussed below), because the poisonous tree doctrine does not apply to Miranda, and
because the gun was in plain view, the seizure of the gun was still lawful.
Dan's motion to suppress the gun is likely to fail.
Statement
94
A statement is obtained in violation of Miranda where an individual is in custody,
and an officer is interrogating the individual without first providing the appropriate
Miranda warnings. Here, it is clear that the officer did not provide Miranda warnings, so
the question is whether Dan was in custody and whether the police officers question as
to whether Dan owned the gun constituted interrogation.
Custody
An individual is in custody for the purposes of Miranda where a reasonable
person in his position would not feel free to leave and end the detention. However, the
supreme court has specifically held that routine traffic stops did not constitute custody
for the purposes of Miranda. In this case, therefore, the routine security checkpoint
would not be considered custody for Miranda purposes. It does not matter that Dan
thought that he had no choice, because the test is objective, and not subjective. When
the police officer asked Dan if he would consent, it is also possible that a reasonable
person in Dan's position would have interpreted this question as indicating that he was
free to not consent.
Because Dan was not in custody at the time that he made the statement, it was
not illicit in violation of Miranda and is admissible.
Interrogation
A police officer is considered to be interrogating an individual where his
questions are reasonably likely to illicit incriminating statements. Here, the officer asked
Dan if he was the owner of the gun. This question does not seem designed to lead to
an incriminating statement, only to determine who was the owner of the gun. In
responding to the question, Dan would have been expected to give a simple yes or no.
In the event of a non, probably a statement about who it belonged to would be
expected. From the perspective of the officer, it probably seemed unlikely that this
question would illicit a confession to the theft of the gun.
Because Dan was not being interrogated at the time he made the statement, it
was not obtained in violation of Miranda for this reason as well. Dan's motion to
suppress the statement is likely to fail.
2. Likelihood of conviction
95
Elements of theft
Larceny, or theft, is the taking or concealing of the property of another with the
intent to permanently deprive the owner or rightful possessor of that property of the
property. The issue here is whether Dan took property that belonged to the church, and
whether he intended to permanently deprive the church of the gun.
Taking
A taking of the property of another occurs where the defendant physically moves
the property of another, or conceals it on his person. In this case, although Dan may
have had a right to possess the gun at the time that the woman handed it to him, it
belonged to the Church as soon as the woman handed it over and told Dan that she
wanted the Church to have it. Although Dan may have intended to give the gun to the
church, a taking of the gun occurred when he did not give it to the church and instead
placed it in his car. When he turned over the book and mislead the church as to the
donation, his right of possession did not continue to exist and his action met the first
element of larceny.
Intent to permanently deprive
A defendant need not have had the intent to permanently deprive the owner or
rightful possessor at the time that the taking of the property occurred. It is enough that
the intent to permanently deprive arose after the taking. In this case, it is not clear if
Dan had the intent to permanently deprive. It would appear that he did not intend to
ever give the gun to the church when he gave them only the book and placed the gun in
his car. This is circumstantial evidence of an intent to permanently deprive and may be
sufficient to meet the requirements for this element. On the other hand, he also told the
officer that he was planning on giving it back. If he merely later changed his mind about
the gun, this would be irrelevant, because if he had the requisite intent even this would
be enough. However, this statement could also be circumstantial evidence indicating
that he never had the required intent. This is a question for the jury to decide,
depending on whether they believe the defendants statements.
Mistake of law
96
Dan appears to believe that he "stole the gun." His beliefs about the illegality of
his actions are immaterial however. His statement would be relevant only to determine
whether he had an intent to permanently deprive. This is because belief that one
completed an unlawful act that is actually lawful does not render the act unlawful.
Embezzlement
Embezzlement is a type of theft, and is the taking of a piece of property that the
defendant had a right to possess at the time of the taking. Therefore, even if Dan had a
right to possess the gun at the time, Dan could still be convicted of embezzlement, as
opposed to basic theft. This conviction would turn on whether the jury found that
placing the gun in the car was sufficient to indicate that Dan intended to convert the
Church's property into his own and permanently deprive the church of it.
Because Dan took a gun that he did not have a right to possess, and because
circumstantial evidence indicates he intended to permanently deprive the church of the
gun, he is likely to be convicted at trial for theft.